MSCAA Questions Flashcards

1
Q

A 24 year old man has poor urine flow and takes a very long time to empty his bladder. He has no other urinary symptoms. He has been well previously apart from one episode of non-gonococcal urethritis 1 year ago.
Which is the most likely diagnosis?
A. Overactive bladder
B. Neurogenic bladder
C. Phimosis
D. Prostatic hypertrophy
E. Urethral stricture

A

E. Urethral stricture

How well did you know this?
1
Not at all
2
3
4
5
Perfectly
2
Q

A 67 year old man is found to have an ejection systolic murmur. He is otherwise well. His pulse rate is 72 bpm and BP 128/84 mmHg. His chest is clear.
Investigations:
ECG shows sinus rhythm.
Echocardiography shows aortic stenosis, valve gradient 50 mmHg. Left ventricular (LV) diastolic dysfunction, LV ejection fraction 45% (>55).
Which is the most appropriate management?
A. Clinical review and echocardiography in 6 months
B. Reassure and discharge
C. Refer for aortic valve replacement
D. Start bisoprolol fumarate and advise review if symptomatic
E. Start lisinopril and advise review if symptomatic

A

C. Refer for aortic valve replacement

How well did you know this?
1
Not at all
2
3
4
5
Perfectly
3
Q

A 27 year old woman has muscle weakness which is worse on exercise. When asked to count from one to 100 her voice progressively becomes weaker. She has bilateral ptosis.
Which pathophysiological process is most likely to be responsible for this disorder?
A. Autoimmunity
B. Genetic disorder
C. Infarction
D. Malignancy
E. Meningeal infection

A

A. Autoimmunity

How well did you know this?
1
Not at all
2
3
4
5
Perfectly
4
Q

A 60 year old man has 6 months of dry cough and increasing shortness of breath on effort. He was previously fit and well, and is a non-smoker.
His temperature is 36.8°C, pulse rate 60 bpm and oxygen saturation 89% breathing air.
He has finger clubbing. Cardiac examination is normal, and chest examination reveals bibasal crepitations.
Which is the most likely diagnosis?
A. Bronchiectasis
B. Extrinsic allergic alveolitis
C. Idiopathic pulmonary fibrosis
D. Lung carcinoma
E. Pulmonary tuberculosis

A

C. Idiopathic pulmonary fibrosis

How well did you know this?
1
Not at all
2
3
4
5
Perfectly
5
Q

A 46 year old man has pain in his left leg and tingling in his left big toe. He developed severe lower back pain 1 week ago and he is unable to walk on his left heel. There is loss of pinprick perception over the left great toe.
Which nerve root is the most likely to have been affected?
A. L1
B. L3
C. L5
D. S1
E. S2

A

C. L5

How well did you know this?
1
Not at all
2
3
4
5
Perfectly
6
Q

65 year old man attends the anticoagulant clinic. He has had a metal mitral valve replacement and atrial fibrillation. He takes warfarin 7 mg daily.
There are no signs of bleeding. His pulse rate is 70 bpm, irregularly irregular, with a mechanical second heart sound. His INR is 5.1.
Which is the most appropriate next step in management?
A. Continue warfarin at lower dose
B. Continue warfarin at same dose
C. Give vitamin K intravenously
D. Give vitamin K orally
E. Withhold warfarin for 2 days then restart at lower dose

A

E. Withhold warfarin for 2 days then restart at lower dose

How well did you know this?
1
Not at all
2
3
4
5
Perfectly
7
Q

A 52 year old man has three days of severe epigastric pain, radiating to his back, but no chest pain. He has vomited several times. He was previously well. He drinks approximately 60 units of alcohol a week and smokes 20 cigarettes per day.
There is epigastric tenderness but his abdomen is not distended, and bowel sounds are present.
Which test would confirm the most likely diagnosis?
A. Abdominal X-ray
B. Gastroduodenoscopy
C. Serum alkaline phosphatase concentration
D. Serum amylase concentration
E. Ultrasound scan of abdomen

A

D. Serum amylase concentration

How well did you know this?
1
Not at all
2
3
4
5
Perfectly
8
Q

A 24 year old man attends the Emergency Department after 2 days of vomiting. He has type 1 diabetes. He is drowsy but maintaining his airway. His pulse rate is 100 bpm, BP 90/60 mmHg, respiratory rate 30 breaths per minute and oxygen saturation 96% breathing air.
Investigations:
Blood capillary glucose 32 mmol/L
Blood capillary ketones 6.2 mmol/L (<0.6) Venous pH 7.15 (7.35–7.45)
Which is the most appropriate initial treatment?
A. Intravenous 0.9% sodium chloride
B. Intravenous 1.26% sodium bicarbonate
C. Intravenous antibiotics
D. Intravenous insulin
E. Subcutaneous insulin

A

A. Intravenous 0.9% sodium chloride

How well did you know this?
1
Not at all
2
3
4
5
Perfectly
9
Q

A 55 year old man is rescued from a collapsed building where he has been trapped for 12 hours without water. His temperature is 35.6°C, pulse rate 100 bpm and BP 90/42 mmHg. His JVP is not visible. His abdomen is non tender.
Investigations:
Haemoglobin 168 g/L (130–175) Sodium 148 mmol/L (135–146) Potassium 6.0 mmol/L (3.5–5.3) Urea 25.1 mmol/L (2.5–7.8) Creatinine 184 μmol/L (60–120) Creatine kinase 840 U/L (25–200)
Which is the most likely cause of this biochemical picture?
A. Bladder outflow obstruction
B. Direct renal trauma
C. Hypovolaemia
D. Rhabdomyolysis
E. Sepsis

A

C. Hypovolaemia

How well did you know this?
1
Not at all
2
3
4
5
Perfectly
10
Q

A 24 year old woman has diarrhoea. She is HIV positive and has been working in Namibia.
Investigation:
Faeces microscopy (following modified Ziehl–Neelsen stain): protozoa
Which is the most likely causative organism?
A. Acanthamoeba
B. Cryptosporidium parvum
C. Entamoeba coli
D. Plasmodium falciparum
E. Schistosoma mansoni

A

B. Cryptosporidium parvum

How well did you know this?
1
Not at all
2
3
4
5
Perfectly
11
Q

A 67 year old man has difficulty chewing and speaking. He underwent carotid surgery 2 days ago.
His tongue deviates to the right when he is asked to protrude it. Which nerve has been damaged?
A. Left glossopharyngeal nerve
B. Left hypoglossal nerve
C. Left vagus nerve
D. Right glossopharyngeal nerve
E. Right hypoglossal nerve

A

E. Right hypoglossal nerve

How well did you know this?
1
Not at all
2
3
4
5
Perfectly
12
Q

An 18 year old woman has had 3 years of intermittent zig-zagging and flashing lights in both eyes associated with headache. These episodes occur 2-3 times per month, last approximately half an hour, and are associated with nausea and vomiting. Her vision is affected at the time of each episode but returns to normal afterwards.
Which is the most likely diagnosis?
A. Acute glaucoma
B. Migraine
C. Occipital lobe epilepsy
D. Retinal detachment
E. Tension-type headache

A

B. Migraine

How well did you know this?
1
Not at all
2
3
4
5
Perfectly
13
Q

A 75 year old man has had 3 days of intermittent headaches, blurred vision and vomiting. For the past 24 hours he has had a severe left sided headache and eye pain, accompanied by blurred vision and vomiting. His left eye is red and the left pupil is dilated.
Which investigation is most likely to confirm the diagnosis?
A. CT scan of head
B. Erythrocyte sedimentation rate
C. Fluorescein staining of the cornea
D. Measurement of intraocular pressure
E. MR scan of head

A

D. Measurement of intraocular pressure

How well did you know this?
1
Not at all
2
3
4
5
Perfectly
14
Q

A 72 year old woman has had 4 months of progressive difficulty walking. She describes numbness and tingling in her feet and has fallen on several occasions.
On examination of her lower limbs, she has normal tone, moderate weakness of ankle dorsiflexion and plantar flexion, normal knee jerks, but absent ankle jerks and extensor plantars. Romberg’s test is positive. She has reduced vibration sense, and joint position sense is impaired up to the ankle joints. Temperature and pinprick sensations are normal.
Which investigation is most likely to confirm the diagnosis?
A. HbA 1c
B. Serum folate
C. Serum protein electrophoresis
D. Serum vitamin B 12
E. Serum vitamin D

A

D. Serum vitamin B 12

How well did you know this?
1
Not at all
2
3
4
5
Perfectly
15
Q

A 34 year old woman has a recurrent itchy rash (see image). She is a firefighter and says that she does not want any treatments that may affect her level of alertness.
Which is the most appropriate treatment to control her symptoms?
A. Oral chlorphenamine maleate
B. Oral loratadine
C. Oral prednisolone
D. Topical aqueous cream
E. Topical hydrocortisone

A

B. Oral loratadine

How well did you know this?
1
Not at all
2
3
4
5
Perfectly
16
Q

A 29 year old woman has 2 days of marked loss of vision and acute pain in her left eye. The pain is worse when she changes her gaze direction.
Her eyes appear normal on general inspection. Her vision is ‘count fingers only’ in the affected eye. The swinging flashlight test shows that the left pupil dilates when a bright light is moved from the right eye to the left eye. The optic discs are normal on fundoscopy.
Which is the most likely diagnosis?
A. Acute closed angle glaucoma
B. Giant cell arteritis
C. Idiopathic intracranial hypertension
D. Migraine with aura
E. Retrobulbar optic neuritis

A

E. Retrobulbar optic neuritis

How well did you know this?
1
Not at all
2
3
4
5
Perfectly
17
Q

45 year old woman develops an intensely painful eruption around her right eye. The illness started with pain 5 days previously, followed by the appearance of a few vesicles, which has now developed into the rash (see image). She has no significant medical history. Treatment is started.
Which is the most likely long-term outcome?
A. Complete resolution with no sequelae
B. Corneal ulceration
C. Extensive scarring of the right temple
D. Partial ptosis
E. Reduced visual acuity

A

A. Complete resolution with no sequelae

How well did you know this?
1
Not at all
2
3
4
5
Perfectly
18
Q

A 48 year old woman has rheumatoid arthritis. She takes regular paracetamol and has no drug allergies. She is due to commence methotrexate weekly.
Which additional treatment should be prescribed?
A. Calcium carbonate
B. Folic acid
C. Pyridoxine hydrochloride
D. Thiamine
E. Vitamin D

A

B. Folic acid

How well did you know this?
1
Not at all
2
3
4
5
Perfectly
19
Q

A 74 year old woman has 6 months of progressive weakness of her right leg and 3 months of a similar problem on the left, resulting in several falls. She has also noticed difficulty using her hands and can no longer fasten the buttons on her clothes.
There is wasting of both legs and the hands, particularly the thenar eminences. There is fasciculation in her right quadriceps. Tone is increased in both legs, with brisk reflexes.
Which is the most likely diagnosis?
A. Chronic inflammatory demyelinating polyneuropathy
B. Motor neurone disease
C. Multiple sclerosis
D. Myasthenia gravis
E. Polymyositis

A

B. Motor neurone disease

How well did you know this?
1
Not at all
2
3
4
5
Perfectly
20
Q

A 61 year old man has had 2 months of ankle swelling. He has hypertension and a 30 year history of seronegative polyarthritis. His medication includes ramipril, sulfasalzine, hydroxychloroquine sulfate and diclofenac.
His BP is 156/90 mmHg. He has pitting oedema to mid thigh and signs of chronic deforming polyarthropathy in his hands, but no joint tenderness. His optic fundi show silver wiring and arteriovenous nipping. Urinalysis: protein 4+, no other abnormalities.
Investigations:
Sodium 133 mmol/L (135–146)
Potassium 5.4 mmol/L (3.5–5.3)
Urea 9.0 mmol/L (2.5–7.8)
Creatinine 119 μmol/L (60–120)
Albumin 21 g/L (35–50)
CRP 43 mg/L (<5)
Urinary protein:creatinine ratio 1100 mg/mmol (<30)
Which is the most appropriate initial treatment?
A. Candesartan cilexetil
B. Furosemide
C. Indapamide
D. Prednisolone
E. Prednisolone and cyclophosphamide

A

B. Furosemide

How well did you know this?
1
Not at all
2
3
4
5
Perfectly
21
Q

A 75 year old woman has had 5 months of a 2 cm red plaque on her leg. Investigation:
Skin biopsy: Bowen’s disease
Which is the most appropriate topical treatment?
A. 5-fluorouracil (Efudix ® ) cream
B. Betamethasone valerate (Betnovate ® ) cream
C. Diclofenac (Solaraze ® ) gel
D. Isotretinoin gel
E. Salicylic acid gel

A

A. 5-fluorouracil (Efudix ® ) cream

How well did you know this?
1
Not at all
2
3
4
5
Perfectly
22
Q

A 32 year old woman has had palpitations and hot flushes for 4 weeks. She has noticed a painless swelling in her neck over the same time and her weight has decreased by 2 kg. She gave birth 4 months ago after a normal pregnancy. She is not breastfeeding.
Her pulse rate is 120 bpm and BP 140/90 mmHg. She is tremulous and restless. She has a large smooth non-tender goitre.
Investigations:
Free T4 35.6 pmol/L (9–25)
Free T3 10.8 pmol/L (4.0–7.2)
TSH <0.01 mU/L (0.3–4.2)
Thyroid peroxidase antibodies >1600 IU/L (<50) Thyroid stimulating antibodies <1.0 IU/L (<1.75)
Which is the most appropriate initial treatment?
A. Carbimazole
B. Propranolol
C. Propylthiouracil
D. Thyroidectomy
E. Thyrotropin alfa

A

B. Propranolol

How well did you know this?
1
Not at all
2
3
4
5
Perfectly
23
Q

An 80 year old man has an ulcer over the left heel and reduced mobility. He has a loss of appetite. He has type 2 diabetes mellitus and has previously had a myocardial infarction.
The ulcer is 3 cm in diameter and deeply penetrating. Sensory testing shows reduced vibration sense but normal sensation to light touch. His Doppler ratio (ankle brachial pressure index) on the left is 0.68 and on the right is 0.98 (normal value 1.00).
Which is the most likely mechanism of his ulcer?
A. Arterial
B. Neuropathic
C. Nutritional
D. Vasculitic
E. Venous

A

A. Arterial

How well did you know this?
1
Not at all
2
3
4
5
Perfectly
24
Q

A 73 year old man is in hospital with a chest infection. He has several episodes of confusion, anxiety and aggression, during which he attempts to leave the hospital. He is convinced he is being ‘spied on’ by the doctors and nurses and insists that ‘cameras have been installed in my room’. These episodes alternate with periods of marked lethargy, which become more pronounced towards the evening.
His temperature is 37.8°C, pulse rate 100 bpm, BP 110/73 mmHg and respiratory rate 12 breaths per minute.
Which is the most likely diagnosis?
A. Alzheimer’s dementia
B. Bipolar disorder
C. Delirium
D. Lewy body dementia
E. Schizophrenia

A

C. Delirium

How well did you know this?
1
Not at all
2
3
4
5
Perfectly
25
Q

A 78 year old woman is admitted to the surgical unit with a suspected vesicocolic fistula. She has a history of hypertension, type 2 diabetes mellitus and angina. She is taking amlodipine, metformin, gliclazide, simvastatin and bisoprolol. Her serum creatinine is 120 μmol/L (60–120). The consultant surgeon requests a CT scan of abdomen with contrast.
Which medication should be stopped before her CT scan?
A. Amlodipine
B. Bisoprolol
C. Gliclazide
D. Metformin
E. Simvastatin

A

D. Metformin

How well did you know this?
1
Not at all
2
3
4
5
Perfectly
26
Q

A 50 year old man has a 3 month history of right loin pain and weight loss. For the past 20 years, he has smoked ten cigarettes per day. His temperature is 37.4°C, pulse rate is 72 bpm and BP is 142/74 mmHg.
Investigations:
Haemoglobin 11.2 g/L (130–175) Platelets 340 × 109/L (150–400)
White cell count 10.1 × 109/L (4.0–11.0) Urinalysis blood 3+
Which is the most likely diagnosis?
A. Benign prostatic hypertrophy
B. Pyelonephritis
C. Renal calculus
D. Renal cancer
E. Urinary tract infection

A

D. Renal cancer

How well did you know this?
1
Not at all
2
3
4
5
Perfectly
27
Q

A 74 year old man with type 2 diabetes has been increasingly unwell and experiencing progressive thirst and nausea for 2 weeks. Initially he described needing to pass urine more frequently than usual, but now he has not passed urine for 24 hours.
He is dehydrated.
Investigations:
Sodium 149 mmol/L (135–146) Potassium 5.2 mmol/L (3.5–5.3) Chloride 101 mmol/L (95–106)
Urea 15.4 mmol/L (2.5–7.8) Creatinine 208 μmol/L (60–120) Fasting glucose 41.7 mmol/L (3.0–6.0)
Which is the calculated serum osmolality?
A. 206.1 mmol/L
B. 255.2 mmol/L
C. 312.3 mmol/L
D. 355.1 mmol/L
E. Impossible to calculate, more information needed

A

D. 355.1 mmol/L

How well did you know this?
1
Not at all
2
3
4
5
Perfectly
28
Q

A 65 year old man receives a renal transplant. He is transferred back to the ward after four hours in recovery.
His pulse is 106 bpm regular, BP 110/70 mmHg and respiratory rate 18 breaths per minute. His chest is clear on auscultation. His urine output has been 15–20 mL per hour while in recovery. Drain output has been 120 mL since surgery.
Investigations:
Haemoglobin 90 g/L (130–175) (preoperative level 103 g/L) Sodium 142 mmol/L (135–146)
Potassium 5.8 mmol/L (3.5–5.3)
Urea 31.9 mmol/L (2.5–7.8)
Creatinine 590 μmol/L (60–120)
Which is the next most appropriate management step?
A. Blood transfusion
B. Fluid challenge
C. Furosemide
D. Haemofiltration
E. Insulin and dextrose infusion

A

B. Fluid challenge

How well did you know this?
1
Not at all
2
3
4
5
Perfectly
29
Q

A 70 year old man has a sharp stabbing pain in his jaw and cheek that lasts for seconds. He reports that the pain is triggered when brushing his teeth, cold wind and touching his face.
Which is the most appropriate treatment?
A. Carbamazepine
B. Indometacin
C. Morphine
D. Prednisolone
E. Pregabalin

A

A. Carbamazepine

How well did you know this?
1
Not at all
2
3
4
5
Perfectly
30
Q

A 64 year old man has developed a tremor in both arms over the last 6 months. It is worse on the right. He also reports difficulty sleeping due to restlessness.
He appears emotionally flat and has a tremor at rest that is alleviated on movement.
Which neurotransmitter is most likely to be deficient?
A. Acetylcholine
B. Dopamine
C. Glycine
D. Norepinephrine (noradrenaline)
E. Serotonin

A

B. Dopamine

How well did you know this?
1
Not at all
2
3
4
5
Perfectly
31
Q

An 85 year old man is admitted from a nursing home with a spreading cellulitis originating from an ulcer over his right ankle. Cultures taken from the ulcer and blood have grown MRSA.
He is mildly confused. His temperature is 39.5°C, pulse rate 96 bpm and BP 114/60 mmHg.
Which is the most appropriate initial antibiotic treatment?
A. Co-amoxiclav
B. Flucloxacillin
C. Meropenem
D. Piperacillin with tazobactam
E. Vancomycin

A

E. Vancomycin

How well did you know this?
1
Not at all
2
3
4
5
Perfectly
32
Q

A 75 year old man is found collapsed at home and is brought to the Emergency Department. He has right sided weakness and reduced consciousness. He has type 2 diabetes mellitus, atrial fibrillation and hypertension. He is taking warfarin.
Investigations: INR 4.6 (<1.4)
CT scan of the head shows a large intracranial haemorrhage.
He is given intravenous vitamin K.
Which is the most appropriate next additional treatment?
A. Cryoprecipitate
B. Fresh frozen plasma
C. Fibrinogen concentrate
D. No additional treatment needed
E. Prothrombin complex concentrate

A

E. Prothrombin complex concentrate

How well did you know this?
1
Not at all
2
3
4
5
Perfectly
33
Q

A 76 year old woman has no energy and reports excessive tiredness for the past 3 weeks. She has lost 5 kg in weight over the past 3 months. She drinks 30 units of alcohol per week.
She is thin and jaundiced. Her temperature is 37.2°C. She has a palpable epigastric mass and 4 cm liver edge.
Which is the most likely diagnosis?
A. Cholangiocarcinoma
B. Cholecystitis
C. Cirrhosis of the liver
D. Hepatocellular carcinoma
E. Pancreatic adenocarcinoma

A

E. Pancreatic adenocarcinoma

How well did you know this?
1
Not at all
2
3
4
5
Perfectly
34
Q

A 76 year old woman with hypertension is taking amlodipine 10 mg daily. A 24 hour BP measurement shows a mean BP of 168/90 mmHg.
Investigations:
Sodium 135 mmol/L (135–146) Potassium 4.0 mmol/L (3.5–5.3) Urea 7 mmol/L (2.5–7.8) Creatinine 100 μmol/L (60–120) eGFR 68 mL/min/1.73 m2(>60)
Urinary albumin : creatinine ratio 50 mg/mmol (<3.5) Which class of antihypertensive should be added?
A. ACE inhibitor
B. Alpha blocker
C. Beta blocker
D. Loop diuretic
E. Thiazide-like diuretic

A

A. ACE inhibitor

How well did you know this?
1
Not at all
2
3
4
5
Perfectly
35
Q

A 70 year old man is an inpatient on the cardiology ward. He has worsening breathlessness that woke him up last night.
His pulse rate is 99 bpm, BP 160/100 mmHg and respiratory rate 20 breaths per minute. Auscultation of the chest reveals bibasal crepitations, and there is dullness to percussion of both bases.
Chest X-ray shows small bilateral pleural effusions with upper lobe blood vessel diversion.
Which is the most appropriate diagnostic investigation?
A. Coronary angiography
B. CT pulmonary angiography
C. ECG
D. Echocardiography
E. Serum D dimer

A

D. Echocardiography

How well did you know this?
1
Not at all
2
3
4
5
Perfectly
36
Q

An 80 year old man has sudden onset of loss of vision in his right eye. He has hypertension and a previous stroke.
His visual acuity is hand movements only in the right eye and 6/9 in left eye. The right eye has an afferent pupillary defect; left eye pupil responses are normal. On fundoscopy there is a red spot at the right macula.
Which is the most likely diagnosis?
A. Anterior ischaemic optic neuropathy
B. Branch retinal vein occlusion
C. Central retinal artery occlusion
D. Macular degeneration
E. Retinal detachment

A

C. Central retinal artery occlusion

How well did you know this?
1
Not at all
2
3
4
5
Perfectly
37
Q

A 72 year old woman has had inability to sleep well for the past 3 years. She gets to sleep by 23:00 but wakes up two or three times in the night and gets up by 07:00. Her husband says that she doesn’t snore. Her BMI is 23 kg/m2. She carries out her normal daytime activities with no daytime somnolence. She is otherwise well. Her MMSE (Mini Mental State Examination) score is 27/30.
Which is the most likely cause of her insomnia?
A. Depression
B. Early stages of dementia
C. Hypomania
D. Normal age related sleep pattern
E. Obstructive sleep apnoea

A

D. Normal age related sleep pattern

How well did you know this?
1
Not at all
2
3
4
5
Perfectly
38
Q

A 35 year old man visits his GP with 3 days of a red, painful left eye with no discharge.
There is a diffuse area of redness in the medial aspect of his left sclera. His pupils and visual acuity are normal.
Which is the most appropriate management?
A. Arrange assessment in emergency eye clinic
B. Prescribe chloramphenicol eye drops
C. Prescribe corticosteroid eye drops
D. Prescribe topical aciclovir
E. Reassure patient that it will resolve spontaneously

A

A. Arrange assessment in emergency eye clinic

How well did you know this?
1
Not at all
2
3
4
5
Perfectly
39
Q

An 18 year old woman is found dead, sitting in front of a gas fire that is still burning. The flue that carries gases away from the fire is found to be blocked.
Which is the principal mechanism of action of the poison involved in her death?
A. Binding to the site on haemoglobin normally occupied by oxygen
B. Converting carbon dioxide to carbonic acid in the cytoplasm of peripheral cells
C. Converting the iron in haem to an iron salt
D. Damaging the lipid bilayer of alveolar pneumocytes
E. Inhibiting cytochrome enzyme systems

A

A. Binding to the site on haemoglobin normally occupied by oxygen

How well did you know this?
1
Not at all
2
3
4
5
Perfectly
40
Q

A 35 year old man with type 1 diabetes mellitus has burning pain in his feet and difficulty sleeping. He has retinopathy and nephropathy.
Investigation: eGFR 28 mL/min/1.73m2(> 60) Which is the most appropriate management?
A. Acupuncture
B. Amitriptyline
C. Duloxetine
D. Physiotherapy
E. Sodium valproate

A

B. Amitriptyline

How well did you know this?
1
Not at all
2
3
4
5
Perfectly
41
Q

A 24 year old woman has tiredness, bloating and weight loss with bouts of offensive smelling diarrhoea. Abdominal examination is normal.
Investigations:
Haemoglobin 10.0 g/L (115–150)
Mean cell volume (MCV) 78 fL (80–96)
Platelets 350 × 109/L (150–400)
Duodenal biopsy shows flattening of villi and increased lymphocytes in the lamina propria and surface epithelium. In addition, there is gross crypt hyperplasia.
Which is the most likely diagnosis?
A. Carcinoid tumour
B. Coeliac disease
C. Collagenous enteropathy
D. Crohn’s disease
E. Pseudomembranous enteropathy

A

B. Coeliac disease

How well did you know this?
1
Not at all
2
3
4
5
Perfectly
42
Q

An 87 year old man develops profuse watery diarrhoea 6 days after admission for an infective exacerbation of COPD. He is currently taking oral co- amoxiclav.
Which is the most likely causative organism?
A. Campylobacter jejuni
B. Clostridioides difficile (Clostridium difficile)
C. Escherichia coli
D. Norovirus
E. Salmonella enteriditis

A

B. Clostridioides difficile (Clostridium difficile)

How well did you know this?
1
Not at all
2
3
4
5
Perfectly
43
Q

A 22 year old woman has worsening shortness of breath and cough productive of four to five tablespoons of sputum per day. She has a history of childhood pneumonia and recurrent chest infections, and coughed up blood on two occasions many years ago.
On auscultation of her chest there are bilateral scattered wheezes and coarse inspiratory crackles.
Which is the most likely diagnosis?
A. Bronchiectasis
B. COPD
C. Lung cancer
D. Pulmonary fibrosis
E. Sarcoidosis

A

A. Bronchiectasis

How well did you know this?
1
Not at all
2
3
4
5
Perfectly
44
Q

A 52 year old woman has had four episodes of severe epigastric pain associated with vomiting over the past 3 months. The episodes occurred after eating and lasted for about 1 hour before complete resolution. She has a history of type 2 diabetes mellitus and takes metformin.
Abdominal examination is normal. Her BMI is 35 kg/m2. Which investigation is most likely to confirm the diagnosis?
A. Helicobacter stool antigen test
B. Liver function tests
C. Plain abdominal X-ray
D. Ultrasonography of abdomen
E. Upper gastrointestinal endoscopy

A

D. Ultrasonography of abdomen

How well did you know this?
1
Not at all
2
3
4
5
Perfectly
45
Q

An 83 year old woman has recurring ‘dizzy spells’. The episodes are associated with transient shaking of her hands that is most noticeable before her lunch and evening meals. She has hypertension and type 2 diabetes mellitus. Her medication includes metformin 1 g twice daily, gliclazide 80 mg twice daily and ramipril 10 mg daily.
Her BP is 138/82 mmHg supine and 130/78 mmHg erect. Her blood capillary glucose is 6 mmol/L.
Investigations:
Sodium 136 mmol/L (135-146)
Potassium 5.0 mmol/L (3.5-5.3)
Urea 3.9 mmol/L (2.5-7.8)
Creatinine 77 μmol/L (60-120)
Glycated haemoglobin 50 mmol/mol (20-42)
Which is the most appropriate therapeutic change?
A. Increase gliclazide dose
B. Increase metformin dose
C. Reduce gliclazide dose
D. Reduce metformin dose
E. Reduce ramipril dose

A

C. Reduce gliclazide dose

How well did you know this?
1
Not at all
2
3
4
5
Perfectly
46
Q

A 76 year old man has progressive breathlessness with right-sided pleuritic chest pain and weight loss over 6 months. He is a retired mechanic and has a 25 pack-year smoking history.
Investigations:
Chest X-ray: marked volume loss in right hemithorax
CT scan of chest: see image
Which is the most likely diagnosis?
A. Asbestosis
B. Chronic hypersensitivity pneumonitis
C. Lung cancer
D. Malignant pleural mesothelioma
E. Tuberculosis

A

D. Malignant pleural mesothelioma

How well did you know this?
1
Not at all
2
3
4
5
Perfectly
47
Q

A 65 year old woman has severe left-sided abdominal pain. Yesterday, she noticed blood mixed in with her stools. There is no history of weight loss.
Her temperature is 37.7°C. She is very tender on palpation in the left lower quadrant. No masses are felt on rectal examination, but there is blood on the glove.
Which is the most likely cause of her symptoms?
A. Angiodysplasia
B. Colorectal cancer
C. Diverticulitis
D. Haemorrhoids
E. Ulcerative colitis

A

C. Diverticulitis

How well did you know this?
1
Not at all
2
3
4
5
Perfectly
48
Q

A 35 year old woman was admitted two days ago after taking 32 paracetamol tablets. She has alcohol use disorder and weighs 40 kg. She has been treated with a full dose of acetylcysteine.
Which investigation best demonstrates restoration of liver synthetic function?
A. Albumin
B. ALT
C. Bilirubin
D. γGT
E. PT

A

E. PT

How well did you know this?
1
Not at all
2
3
4
5
Perfectly
49
Q

A 64 year old woman has thumping palpitations and says that she sometimes feels her heart ‘gives a sudden jump’. Her husband recently died due to myocardial infarction.
Her pulse rate is 70 bpm, BP 136/80 mmHg. Her heart sounds are normal. Her 12-lead ECG is shown in the tracing.
Which is the most likely cause of her palpitations?
A. Atrial fibrillation
B. Premature supraventricular beats
C. Premature ventricular beats
D. Sinus arrhythmia
E. Ventricular tachycardia

A

C. Premature ventricular beats

How well did you know this?
1
Not at all
2
3
4
5
Perfectly
50
Q

A 35 year old man has painless swelling of the right side of his scrotum. The swelling is soft and fluctuant, and transilluminates.
Which is the most likely diagnosis?
A. Hydrocoele
B. Inguinal hernia
C. Testicular torsion
D. Testicular tumour
E. Varicocoele

A

A. Hydrocoele

How well did you know this?
1
Not at all
2
3
4
5
Perfectly
51
Q

A 67 year old woman has an ulcer with a raised white margin on her left ear; it has been present for 3 years, growing slowly and never completely healing. She spent 20 years living in Australia before returning to the UK recently.
On examination, she has a small ulcerated area, 4 mm × 6 mm, on her left pinna.
Which is the most likely diagnosis?
A. Actinic keratosis
B. Basal cell carcinoma
C. Malignant melanoma
D. Seborrhoeic keratosis
E. Squamous cell carcinoma

A

B. Basal cell carcinoma

How well did you know this?
1
Not at all
2
3
4
5
Perfectly
52
Q

A 90 year old man has had 3 days of spasmodic suprapubic pain radiating to the tip of the penis. His long-term urinary catheter has recently been changed. He is mildly confused.
His temperature is 38.2o C, pulse rate 88 bpm, BP 146/88 mmHg, respiratory rate 15 breaths per minute and oxygen saturation 96% breathing
air. Urinalysis: dark and strong smelling, protein 1+, blood 1+, negative for leucocytes and nitrites.
Which factor(s) indicate(s) the need to start antibiotics?
A. Blood and protein in urine
B. Dark strong-smelling urine
C. Fever and mild confusion
D. Presence of pain
E. Recent catheter change

A

C. Fever and mild confusion

How well did you know this?
1
Not at all
2
3
4
5
Perfectly
53
Q

A 72 year old man with COPD has increased production of purulent sputum and dyspnoea. He has a poor appetite and can rarely finish meals.
Which is the most appropriate method of nutritional assessment?
A. Malnutrition Universal Screening Tool (MUST)
B. Mini Nutritional Assessment (MNA)
C. Serum albumin concentration
D. Skinfold thickness
E. Subjective Global Assessment (SGA)

A

A. Malnutrition Universal Screening Tool (MUST)

How well did you know this?
1
Not at all
2
3
4
5
Perfectly
54
Q

A 48 year old man has visible haematuria and right loin pain. His temperature is 37.3°C, pulse rate 72 bpm and BP 170/97 mmHg. Masses are palpable in both flanks.
Investigations:
Creatinine 220 μmol/L (60-120) Urinalysis: blood 4+
Which is the most appropriate next investigation?
A. CT scan of kidneys, ureters and bladder
B. Cystoscopy
C. MR scan of renal tract
D. Ultrasound scan of renal tract
E. Urine cytology

A

D. Ultrasound scan of renal tract

How well did you know this?
1
Not at all
2
3
4
5
Perfectly
55
Q

A 35 year old man visits his GP with a severe frontal headache of 12 hours’ duration. It started suddenly, reaching maximum intensity within 1 minute. He has associated nausea.
At the onset of his headache he noticed a small hole in his vision. This hole started centrally, moved to the edge of his vision and has now resolved. The headache is worse in bright light. Paracetamol has not helped his pain.
Which feature should prompt immediate referral to hospital?
A. Abrupt onset
B. Failure to respond to paracetamol
C. Nausea
D. Photophobia
E. Visual disturbance

A

A. Abrupt onset

How well did you know this?
1
Not at all
2
3
4
5
Perfectly
56
Q

A 28 year old man has an insurance medical.
His pulse rate is 72 bpm and BP 210/110 mmHg. There is radiofemoral
delay. A systolic murmur is audible on auscultation.
Chest X-ray reveals notching of the ribs in the mid-clavicular line. Which is the most likely diagnosis?
A. Coarctation of the aorta
B. Dissecting aortic aneurysm
C. Marfan’s syndrome
D. Renal artery stenosis
E. Takayasu’s arteritis

A

A. Coarctation of the aorta

How well did you know this?
1
Not at all
2
3
4
5
Perfectly
57
Q

A 42 year old woman has two episodes of haemoptysis. She also reports two months of sinusitis with pain, stuffiness and nose bleeds. She has some loosening of her teeth and painful oral ulceration. Previously, she noticed a rash and nodules over her elbows but they have disappeared.
Investigations:
Chest X-ray: see image
Urinalysis: occasional red cell casts
Which is the most likely diagnosis?
A. Behçet’s disease
B. Granulomatosis with polyangiitis
C. Metastatic nasopharyngeal carcinoma
D. Syphilis
E. Tuberculosis

A

B. Granulomatosis with polyangiitis

How well did you know this?
1
Not at all
2
3
4
5
Perfectly
58
Q

A 19 year old man has had a sore throat, malaise and intermittent fever for 2 weeks.
His pulse rate is 74 bpm and BP 115/75 mmHg. His throat is red and tonsils are swollen. His sclerae are yellow-tinged. There are multiple soft palpable lymph nodes in the neck. There is tenderness in the right upper abdominal quadrant.
Which is the most appropriate diagnostic investigation?
A. Epstein–Barr virus test
B. HIV test
C. Lymph node biopsy for histology
D. Mantoux test
E. Throat swab and culture

A

A. Epstein–Barr virus test

How well did you know this?
1
Not at all
2
3
4
5
Perfectly
59
Q

A 27 year old woman has had abdominal pain for 48 hours. She also reports recurrent mouth ulcers and altered bowel habit for a few weeks.
Her temperature is 37.5°C. She has central abdominal tenderness.
Investigations:
CT colonoscopy shows a normal appendix with distal small bowel thickening. There are enlarged nodes in the small bowel mesentery.
Which is the most likely diagnosis?
A. Crohn’s ileitis
B. Intestinal tuberculosis
C. Meckel’s diverticulitis
D. Mesenteric adenitis
E. Small bowel lymphoma

A

A. Crohn’s ileitis

How well did you know this?
1
Not at all
2
3
4
5
Perfectly
60
Q

A 34 year old man has cough and weight loss.
A diagnosis of tuberculosis is confirmed and treatment is started. As part of
his monitoring, he is screened for loss of visual acuity. Which antituberculosis drug is an indication for visual monitoring?
A. Ethambutol hydrochloride
B. Isoniazid
C. Moxifloxacin
D. Pyrazinamide
E. Rifampicin

A

A. Ethambutol hydrochloride

How well did you know this?
1
Not at all
2
3
4
5
Perfectly
61
Q

A 21 year old woman presents to her GP with moderately severe acne. She has tried topical retinoids and topical antibiotics without satisfactory response. She has a history of a deep vein thrombosis following a long-haul flight.
Which is the most appropriate treatment?
A. Co-cyprindiol
B. Desogestrel
C. Flucloxacillin
D. Isotretinoin
E. Lymecycline

A

E. Lymecycline

How well did you know this?
1
Not at all
2
3
4
5
Perfectly
62
Q

A 45 year old man has had weight loss, fatigue and polyuria for 3 months. He takes a number of multivitamin preparations.
Clinical examination is unremarkable.
Investigations:
Serum corrected calcium 2.9 mmol/L (2.2–2.6) Phosphate 0.82 mmol/L (0.8–1.5)
Serum alkaline phosphatase 154 IU/L (25–115) Parathyroid hormone 7.9 pmol/L (1.6–8.5)
Serum electrolytes and urea are normal. Which is the most likely diagnosis?
A. Bony metastases
B. Excess calcium intake
C. Primary hyperparathyroidism
D. Sarcoidosis
E. Vitamin D excess

A

C. Primary hyperparathyroidism

How well did you know this?
1
Not at all
2
3
4
5
Perfectly
63
Q

A 59 year old man has a 1 year history of erectile dysfunction. He has a history of angina, type 2 diabetes mellitus and peripheral vascular disease. He had a thyroidectomy 2 years ago for thyrotoxicosis. His regular medications are aspirin, diltiazem, levothyroxine, metformin, ramipril and simvastatin.
His BP is 140/90 mmHg lying and 135/85 mmHg standing. His foot pulses are not palpable. He has normal sensation in his feet.
Which is most likely to be the main cause of his erectile dysfunction?
A. Adverse effect of medication
B. Autonomic neuropathy
C. Hypothyroidism
D. Testosterone deficiency
E. Vascular insufficiency

A

E. Vascular insufficiency

How well did you know this?
1
Not at all
2
3
4
5
Perfectly
64
Q

A 52 year old man has had 3 months of fatigue. He has a history of ulcerative colitis and takes mesalazine. He drinks 20 units of alcohol per week. His temperature is 36.8°C and pulse rate 80 bpm. He has 3 cm hepatomegaly.
Investigations:
Albumin 36 g/L (35–50) ALT 65 IU/L (10–50) ALP 580 IU/L (25–115) Bilirubin 18 μmol/L (<17) γGT 230 IU/L (9–40)
Which is the most likely diagnosis?
A. Alcoholic hepatitis
B. Cholangiocarcinoma
C. Choledocholithiasis
D. Hepatocellular carcinoma
E. Primary sclerosing cholangitis

A

E. Primary sclerosing cholangitis

How well did you know this?
1
Not at all
2
3
4
5
Perfectly
65
Q

A 73 year old man has increasing breathlessness over 1 week. He has a history of chronic kidney disease and ischaemic heart disease. He takes alfacalcidol, aspirin, atorvastatin, bisoprolol fumarate, furosemide and irbesartan.
There are bibasal inspiratory crepitations and mild peripheral oedema.
Investigations:
Sodium 134 mmol/L (135–146) Potassium 6.7 mmol/L (3.5–5.3) Urea19 mmol/L (2.5–7.8) Creatinine 259 μmol/L (60–120) eGFR 23 mL/min/1.73 m2 (>60)
Which drug is most likely to be contributing to his hyperkalaemia?
A. Alfacalcidol
B. Aspirin
C. Bisoprolol fumarate
D. Furosemide
E. Irbesartan

A

E. Irbesartan

How well did you know this?
1
Not at all
2
3
4
5
Perfectly
66
Q

A 65 year old man is invited to the abdominal aortic aneurysm screening programme. An ultrasound scan shows his abdominal aorta to be 33 mm in diameter.
Which is the most appropriate management plan?
A. Reassure and discharge
B. Refer for angiography
C. Refer for vascular surgery
D. Repeat ultrasound scan in 12 months
E. Request CT scan of abdomen

A

D. Repeat ultrasound scan in 12 months

How well did you know this?
1
Not at all
2
3
4
5
Perfectly
67
Q

A 28 year old woman has pain on swallowing. She has a history of asthma that is well controlled using metered dose salbutamol and beclometasone dipropionate (800 micrograms/day) inhalers.
She has white plaques in her mouth.
An anti-fungal oral suspension is prescribed.
What is the most appropriate management with regard to her beclometasone?
A. Change beclometasone dipropionate to a dry powder formulation B. Change beclometasone dipropionate to fluticasone
C. Change beclometasone dipriopionate to salmeterol
D. Take beclometasone dipropionate using a large volume spacer
E. Take salbutamol and beclometasone dipropionate at least 1 h apart

A

D. Take beclometasone dipropionate using a large volume spacer

How well did you know this?
1
Not at all
2
3
4
5
Perfectly
68
Q

A 37 year old man has increasing fever and shortness of breath. He was admitted 3 days ago with left lower lobe pneumonia due penicillin-sensitive Streptococcus pneumoniae. He is receiving intravenous benzylpenicillin 1.2 g four times daily.
His temperature is 38.5°C, pulse 100 bpm and BP 122/80 mmHg.
Investigations:
Chest X-ray: left basal effusion.
Diagnostic pleural aspiration: Purulent fluid. Microscopy shows numerous polymorphs and Gram-positive cocci
Which is the most appropriate next step in management?
A. Catheter thoracostomy drainage
B. Increase dose of benzylpenicillin
C. Open thoracostomy
D. Switch benzylpenicillin to ceftriaxone
E. Switch benzylpenicillin to vancomycin

A

A. Catheter thoracostomy drainage

How well did you know this?
1
Not at all
2
3
4
5
Perfectly
69
Q

A 52 year old man has 4 weeks of joint pain, fever and weight loss. He is a non-smoker and has no significant medical history.
Examination is unremarkable.
Investigations:
Calcium 3.12 mmol/L (2.2–2.6)
Plasma parathyroid hormone <0.5 pmol/L (0.9–5.4)
Chest X-ray shows perihilar lymphadenopathy. Which is the most likely diagnosis?
A. Granulomatosis with polyangiitis
B. Hodgkin’s lymphoma
C. Primary hyperparathyroidism
D. Sarcoidosis
E. Tuberculosis

A

D. Sarcoidosis

How well did you know this?
1
Not at all
2
3
4
5
Perfectly
70
Q

A 56 year old man has had a single episode of painless visible haematuria. He has no other urinary symptoms and is otherwise fit and well. He has smoked ten cigarettes per day for the past 35 years.
He has a BP of 140/85 mmHg. Urinalysis performed after this episode shows blood 2+, no protein and no nitrites.
Investigations:
Urea 6.5 mmol/L (2.5–7.8) Creatinine 95 μmol/L (60–120)
Urine culture: no growth
Which investigation is most likely to confirm the diagnosis?
A. Flexible cystoscopy
B. Serum prostate specific antigen
C. Transrectal ultrasound scan of prostate
D. Ultrasound scan of kidneys
E. Urine cytology

A

A. Flexible cystoscopy

How well did you know this?
1
Not at all
2
3
4
5
Perfectly
71
Q

The daughter of a 78 year old man is concerned about an area of redness on her father’s back. He lives alone and spends most of his day sitting in a chair.
There is a reddened area over the sacrum, but his skin is intact.
He is considered to be at risk of pressure ulcers.
Which member of the community multidisciplinary team would be most suited to conduct an initial assessment?
A. Dietician
B. District nurse
C. Occupational therapist D. Physiotherapist
E. Tissue viability nurse

A

B. District nurse

How well did you know this?
1
Not at all
2
3
4
5
Perfectly
72
Q

A junior doctor sustains a needle-stick injury while inserting a venous cannula into a 28 year old man. The patient has a history of intravenous drug use.
Which is the most important first action?
A. Encourage bleeding from the wound
B. Make an appointment with occupational health
C. Seek post-exposure prophylaxis
D. Test the patient for HIV
E. Wash the wound with alcohol

A

A. Encourage bleeding from the wound

How well did you know this?
1
Not at all
2
3
4
5
Perfectly
73
Q

A 32 year old woman has 3 weeks of fever, rigors and lethargy. In the past week, she has also become breathless on exertion. She is an intravenous drug user.
Her temperature is 38°C, pulse rate 100 bpm regular, and BP 100/60 mmHg. Her JVP is raised with predominant V waves. There is a pansystolic murmur at the left sternal edge on inspiration. She has reduced air entry with dullness to percussion at the right lung base. She has swelling of both ankles.
Which is the most likely pathogen?
A. Enterococcus faecalis
B. Staphylococcus aureus
C. Staphylococcus epidermidis
D. Streptococcus bovis / streptococcus equinus complex
E. Streptococcus viridans

A

B. Staphylococcus aureus

How well did you know this?
1
Not at all
2
3
4
5
Perfectly
74
Q

A 34 year old woman has sudden onset of right arm weakness and inability to speak. She has a history of migraine and generalised joint pains. Four years ago, she had a deep vein thrombosis in her right leg.
Her pulse rate is 68 bpm and BP 178/94 mmHg. She has an expressive dysphasia. She has flaccid weakness of her right arm and facial droop on the right lower half of her face.
Investigations:
Haemoglobin 118 g/L (115–150) White cell count 4.3 × 109/L (3.8–10.0) Neutrophils 2.1 × 109/L (2.0–7.5) Lymphocytes 0.6 × 109/L (1.1–3.3) Platelets 132 × 109/L (150–400)
Total cholesterol 4.6 mmol/L (<5.0)
CT scan of head left frontoparietal infarct
Which additional investigation is most likely to reveal the underlying cause of her stroke?
A. Anti-dsDNA antibody
B. Anticardiolipin antibody C. Anti-Ro antibody
D. Rheumatoid factor
E. Serum immunoglobulins

A

B. Anticardiolipin antibody

How well did you know this?
1
Not at all
2
3
4
5
Perfectly
75
Q

A 55 year old woman has been feeling tired and sleepy. Her partner says that she snores heavily. She has a history of type 2 diabetes mellitus and takes metformin.
Her BMI is 38 kg/m2. Her oxygen saturation is 95% breathing air. Her Epworth sleepiness score is 19 (normal <11). Her HbA1cis 60 mmol/mol (20-42).
Which treatment is most likely to improve her daytime somnolence?
A. Bariatric surgery
B. Continuous positive airway pressure ventilation
C. Long acting insulin
D. Mandibular advancement device
E. Modafinil

A

B. Continuous positive airway pressure ventilation

How well did you know this?
1
Not at all
2
3
4
5
Perfectly
76
Q

A 72 year old man has had six months of increased frequency of defaecation and three months of dark red rectal bleeding mixed with the stool.
Investigations:
Haemoglobin 101 g/L (130–175)
Mean cell haemoglobin (MCH) 24 pg (27–33) MCV 73 fL (80–96)
White cell count 9.1 x 109/L (3.0–10.0) Platelets 354 x 109/L (150–400)
Which is the most likely diagnosis?
A. Colonic carcinoma
B. Diverticular disease
C. Haemorrhoids
D. Ischaemic colitis
E. Ulcerative colitis

A

A. Colonic carcinoma

How well did you know this?
1
Not at all
2
3
4
5
Perfectly
77
Q

A 24 year old man is reviewed following a recent orchidectomy. The pathologist’s report describes a mass with cystic spaces. Histological examination shows areas of mature cartilage and columnar epithelium.
Which is the most likely diagnosis?
A. Chondrosarcoma
B. Hamartoma
C. Lymphoma
D. Seminoma
E. Teratoma

A

E. Teratoma

How well did you know this?
1
Not at all
2
3
4
5
Perfectly
78
Q

A 50 year old man has had increasing breathlessness when climbing the stairs. There is no chest pain, wheeze or cough. He has COPD. He takes a salbutamol inhaler several times per day. He is an ex-smoker of 3 months and has a 30 pack-year smoking history. His weight is stable and he is otherwise well.
Investigation:
FEV1 : 75% predicted
Which is the most appropriate next step in pharmacological management?
A. Inhaled beclometasone
B. Inhaled tiotropium and salmeterol
C. Nebulised salbultamol and ipratropium bromide
D. Oral prednisolone
E. Oral theophylline

A

B. Inhaled tiotropium and salmeterol

How well did you know this?
1
Not at all
2
3
4
5
Perfectly
79
Q

An 86 year old woman has had three falls in the past 3 months. On each occasion, she describes feeling lightheaded and dizzy prior to falling. She is taking alendronic acid, amlodipine, atorvastatin, metformin and zolpidem tartrate. Her BP is 132/80 mmHg sitting and 138/84 mmHg standing.
Which medication is most likely to be contributing to her falls?
A. Alendronic acid
B. Amlodipine
C. Atorvastatin
D. Metformin
E. Zolpidem tartrate

A

E. Zolpidem tartrate

How well did you know this?
1
Not at all
2
3
4
5
Perfectly
80
Q

A 81 year old man has three months of malaise, bleeding gums and pain in his legs. He has been struggling to look after himself since his wife died one year ago. He has a poor diet and reports that he mostly has tea and toast.
He has gingival hypertrophy and skin petechiae. Which micronutrient deficiency is he most likely to have?
A. Magnesium
B. Vitamin A
C. Vitamin B 1
D. Vitamin C
E. Zinc

A

D. Vitamin C

How well did you know this?
1
Not at all
2
3
4
5
Perfectly
81
Q

A 45 year old man has had 6 months of tiredness, reduced libido and erectile dysfunction.
Investigations:
Testosterone 1.8 nmol/L (9.9–27.8) LH 1.2 U/L (1–8)
FSH 1.0 U/L (1–12)
Which is the most likely cause of his presentation?
A. Anabolic steroid misuse
B. Androgen insensitivity syndrome
C. Congenital adrenal hyperplasia
D. Klinefelter’s syndrome
E. Pituitary adenoma

A

E. Pituitary adenoma

How well did you know this?
1
Not at all
2
3
4
5
Perfectly
82
Q

A 24 year old man is admitted to hospital with an exacerbation of asthma. His symptoms improve with treatment, and he is ready for discharge after 24 hours. His discharge medication includes a salbutamol inhaler, a combined beclometasone and salmeterol inhaler, and a short course of oral prednisolone.
Which further management must be provided prior to discharge?
A. Antibiotic rescue pack
B. Course of antihistamines
C. Nebuliser machine for use at home
D. Personalised asthma action plan
E. Volumatic spacer device

A

D. Personalised asthma action plan

How well did you know this?
1
Not at all
2
3
4
5
Perfectly
83
Q

A 40 year old man develops sudden breathlessness 5 days after an acute inferior ST-elevation myocardial infarction treated by primary coronary intervention.
His pulse rate is 110 bpm, BP 110/75 mmHg, respiratory rate 22 breaths per minute and oxygen saturation 92% on breathing 28% oxygen. There is a pansystolic murmur at the apex and bibasal inspiratory crackles.
Which is the most likely cause of this presentation?
A. Acute pulmonary embolus
B. Aortic regurgitation
C. Cardiac tamponade
D. Papillary muscle rupture
E. Pericarditis

A

D. Papillary muscle rupture

How well did you know this?
1
Not at all
2
3
4
5
Perfectly
84
Q

A 48 year old man presents to his doctor with headaches. He has noticed that his hands have become larger, and his facial features have coarsened. More recently, his vision has deteriorated. He has an upper temporal defect in both visual fields.
Damage to which structure is the most likely source of his visual problems?
A. Lateral geniculate body B. Occipital cortex
C. Oculomotor nerve
D. Optic chiasm
E. Optic radiation

A

D. Optic chiasm

How well did you know this?
1
Not at all
2
3
4
5
Perfectly
85
Q

A 42 year old man has a rash on his face, mainly around his chin. The rash started 24 hours ago with a 0.5 cm thin-walled blister that then ruptured, leaving a yellow crusted lesion that has since enlarged and now other similar lesions are appearing in the same area. He is a primary school teacher.
Which is the most likely causative organism?
A. Escherichia coli
B. Pseudomonas aeruginosa
C. Staphylococcus aureus
D. Streptococcus pyogenes
E. Varicella zoster virus

A

C. Staphylococcus aureus

How well did you know this?
1
Not at all
2
3
4
5
Perfectly
86
Q

A 48 year old woman has had 3 years of increasing knee pain and reduced physical activity. She has radiologically-confirmed osteoarthritis. She has hypertension and type 2 diabetes. She takes lisinopril, metformin, semaglutide and simvastatin. Her BMI is 48 kg/m2and has not changed despite lifestyle advice and a low calorie diet for the last year.
Investigations:
Glycated haemoglobin 55 mmol/mol (20-42)
Which is the most appropriate management?
A. Intensify lifestyle measures and review in 6 months
B. Prescribe orlistat
C. Refer for bariatric surgery
D. Refer for bilateral knee replacements
E. Start insulin therapy

A

C. Refer for bariatric surgery

How well did you know this?
1
Not at all
2
3
4
5
Perfectly
87
Q

A 22 year old soldier steps off a cramped military aircraft following a long flight from the United Kingdom. She suddenly collapses and hits her head on the ground. While unconscious, she has asynchronous jerking of her limbs for less than 15 seconds. Witnesses say that she looked pale. She regains consciousness within 1 minute.
What is the most likely cause of her collapse?
A. Cardiac arrhythmia B. Epilepsy
C. Hypoglycaemia
D. Pulmonary embolism E. Vasovagal syncope

A

E. Vasovagal syncope

How well did you know this?
1
Not at all
2
3
4
5
Perfectly
88
Q

A 60 year old man has had malaise and weight loss for 3 months and during the past week has been coughing up blood.
Investigation:
Chest X-ray: left upper zone consolidation with cavitation
Which is the most appropriate next diagnostic investigation?
A. Bronchoscopy and lavage
B. Interferon-γ release assay (IGRA)
C. Mantoux test
D. Sputum cytology
E. Sputum microscopy

A

E. Sputum microscopy

How well did you know this?
1
Not at all
2
3
4
5
Perfectly
89
Q

A 55 year old man is referred to the vascular outpatient clinic with bilateral claudication, limiting his walking distance to 10 metres. He is a smoker.
Imaging shows chronic distal aortic and bilateral common iliac occlusive disease.
Which is the most appropriate surgical intervention?
A. Aortic endarterectomy
B. Aorto-bifemoral bypass graft
C. Aorto-iliac embolectomy
D. Bilateral iliac angioplasty
E. Femoral-to-femoral crossover graft

A

B. Aorto-bifemoral bypass graft

How well did you know this?
1
Not at all
2
3
4
5
Perfectly
90
Q

A 65 year old man sees his GP for monitoring of hypertension and ischaemic heart disease. His medication includes aspirin, atenolol, amlodipine, lisinopril and simvastatin.
He has marked ankle swelling.
Which drug is the most likely cause of his ankle swelling?
A. Amlodipine
B. Aspirin
C. Atenolol
D. Lisinopril
E. Simvastatin

A

A. Amlodipine

How well did you know this?
1
Not at all
2
3
4
5
Perfectly
91
Q

An 85 year old woman was admitted with a stroke three weeks ago. She has a history of urinary incontinence and has a long-term urinary catheter in place. Her current medication includes clopidogrel and ramipril.
Her temperature is 36.8°C, pulse rate 85 bpm and BP 134/74 mmHg. A catheter specimen of urine shows >105CFU/mL, mixed growth.
Which is the most appropriate management?
A. No change in treatment
B. Remove urinary catheter
C. Request antibiotic sensitivities
D. Start oral ciprofloxacin
E. Start oral trimethoprim

A

A. No change in treatment

How well did you know this?
1
Not at all
2
3
4
5
Perfectly
92
Q

A 28 year old man has a headache, intermittent fever, sore throat and diarrhoea.
His temperature is 37.7°C. His fauces are red and there are two small aphthous ulcers on his left buccal mucosa. He also has a maculopapular erythematous rash on his upper trunk, red hands and folliculitis on his chest. His liver and spleen are just palpable and he has mild neck stiffness.
Investigations:
Haemoglobin 135 g/L (130–175) White cell count 3.3 x 109/L (3.0–10.0) Platelets 84 x 109/L (150–400)
Which investigation is most likely to lead to a diagnosis?
A. First catch urine microscopy
B. Glandular fever screening test
C. HIV serology
D. Serum antinuclear antibodies
E. Serum toxoplasma gondii IgM antibody titre

A

C. HIV serology

How well did you know this?
1
Not at all
2
3
4
5
Perfectly
93
Q

A 79 year old woman has six months of increasing breathlessness on exertion.
Her pulse rate is 72 bpm, irregularly irregular, and BP 118/72 mmHg. She has a diastolic murmur best heard at the apex in expiration.
Which is the most likely cause of her murmur?
A. Aortic regurgitation
B. Aortic stenosis
C. Hypertrophic cardiomyopathy
D. Mitral regurgitation
E. Mitral stenosis

A

E. Mitral stenosis

How well did you know this?
1
Not at all
2
3
4
5
Perfectly
94
Q

A 78 year old woman is found dead at home. At autopsy, the pathologist finds bilateral pneumonia and meningitis. Microscopy of a meningeal swab shows Gram-positive cocci arranged in pairs.
Which is the most likely causative organism?
A. Candida albicans
B. Neisseria meningitidis
C. Pseudomonas aeruginosa
D. Staphylococcus aureus
E. Streptococcus pneumoniae

A

E. Streptococcus pneumoniae

How well did you know this?
1
Not at all
2
3
4
5
Perfectly
95
Q

A 40 year old man was admitted to hospital with central crushing chest pain. Examination revealed pale cream coloured nodules on both elbows and
medial aspects of his upper eyelids.
ECG on admission showed ST elevation and T wave inversion.
He deteriorated and died.
A post mortem examination is performed and shows very severe narrowing of the anterior descending branch of the left coronary artery.
Which is the most likely causative mechanism?
A. Atheroma
B. Arterial dissection
C. Malignant deposit
D. Thrombosis
E. Vasculitis

A

A. Atheroma

How well did you know this?
1
Not at all
2
3
4
5
Perfectly
96
Q

A 55 year old man visits his GP concerned that he may be a carrier of cystic fibrosis, a condition that has just been diagnosed in his 5 year old grandson. He has heard that this is an inherited condition, but no one else in his family has the illness.
What is the likelihood that the grandfather is a carrier?
A. 1 in 2 B. 1 in 4 C. 1 in 8 D. 1in16 E. 1in25

A

A. 1 in 2

How well did you know this?
1
Not at all
2
3
4
5
Perfectly
97
Q

A 79 year old woman has been repeatedly found wandering at night by her neighbours. The problem has progressively worsened over 6 months. She is independent in her activities of daily living, although her family do her shopping. She was previously well.
What aspect of cognition is likely to show the greatest impairment?
A. Attention
B. Concentration
C. Praxis
D. Registration of information E. Short-term memory

A

E. Short-term memory

How well did you know this?
1
Not at all
2
3
4
5
Perfectly
98
Q

A 52 year old woman reports increased urinary frequency, urgency and urge incontinence. She has multiple sclerosis, which affects her walking. A midstream urine sample shows no cells and is sterile on culture. A bladder scan shows a residual volume of 300 mL. Urodynamic assessment shows that she has a neuropathic bladder.
Which is the most appropriate management?
A. α-Adrenoceptor blocker
B. Anticholinergic drug
C. Indwelling urethral catheter
D. Intermittent self catheterisation
E. Suprapubic catheter

A

D. Intermittent self catheterisation

How well did you know this?
1
Not at all
2
3
4
5
Perfectly
99
Q

A 43 year old woman is admitted with acute right upper quadrant pain, which radiates to her right shoulder.
Her temperature is 38.6°C and respiratory rate 20 breaths per minute. She is tender to palpation in the right upper quadrant but has no rebound tenderness.
Investigations:
Haemoglobin 132 g/L (115–150)
White cell count 13 x 109/L (3.8–10.0)
Platelets 340 x 109/L (150–400)
Bilirubin 30 μmol/L (<17)
Alanine aminotransferase (ALT) 80 IU/L (10–50) Alkaline phosphatase 306 IU/L (25–115)
Which is the next most appropriate radiological test?
A. Abdominal X-ray
B. CT scan of abdomen
C. Erect chest X-ray
D. MR scan of abdomen
E. Ultrasound scan of abdomen

A

E. Ultrasound scan of abdomen

How well did you know this?
1
Not at all
2
3
4
5
Perfectly
100
Q

An 84 year old man develops profuse diarrhoea whilst in hospital. An outbreak of Clostridioides (Clostridium) difficile has occurred in his ward.
Which feature of this organism makes it particularly difficult to destroy?
A. Motility
B. Outer capsule
C. Rapid mutation
D. Spore formation
E. Surface adherence

A

D. Spore formation

How well did you know this?
1
Not at all
2
3
4
5
Perfectly
101
Q

A48yearoldmanhas1dayofsevererightupperquadrantpain.Hehas vomited five times. He smokes 10 cigarettes per day and drinks 31 units of alcohol per week. He is tender in the epigastrium and right upper quadrant, and there is voluntary guarding.
His temperature is 37.8°C, pulse rate 90 bpm and BP 140/84 mmHg. He is tender in the epigastrium and right upper quadrant, and there is voluntary guarding.
Investigations:
White cell count
ALT
Alkaline phosphatase Bilirubin
Amylase CRP
15 × 109/L 41 IU/L
125 IU/L 14 μmol/L 222U/L 42 mg/L
(3.8–10.0) (10–50)
(25–115) (<17)
(<220) (<5)
Which is the most likely diagnosis?
A. Acute cholangitis B. Acute cholecystitis C. Acute hepatitis
D. Acute pancreatitis E. Biliary colic

A

B. Acute cholecystitis

How well did you know this?
1
Not at all
2
3
4
5
Perfectly
102
Q

A65yearoldwomanhasaweekofdisorientationanddizziness.Shealsohas headaches that are worse when bending over and associated with vomiting. She had a non-small cell lung cancer that was treated with radical radiotherapy
two years ago.
BP is 178/95 mmHg. She has no focal neurological signs. Which is the most likely diagnosis?
A. Cerebral metastases
B. Hypercalcaemia
C. Hyponatraemia
D. Paraneoplastic encephalitis E. Severe hypertension

A

A. Cerebral metastases

How well did you know this?
1
Not at all
2
3
4
5
Perfectly
103
Q

An66yearoldwomanwithadvancedbowelcanceristakingmorphinesulfate (modified release, 30 mg twice daily) for abdominal pain. She is in the last days of life and is now unable to swallow oral medications. She has not needed any doses for breakthrough pain.
Investigations:
eGFR 51 mL/min/1.73 m2(>60)
A continuous subcutaneous infusion of morphine sulfate is required, as a direct conversion from oral morphine sulfate.
Which is the correct dose of morphine sulfate to prescribe over 24 hours?
A. 15 mg
B. 30 mg
C. 60 mg
D. 90 mg
E. 120 mg

A

B. 30 mg

How well did you know this?
1
Not at all
2
3
4
5
Perfectly
104
Q

A24yearoldmandevelopslowbackpainthedayafterfallingwhileplaying tennis. He is usually well and takes no regular medication. He is a laboratory technician.
Which is the most appropriate advice?
A. Avoid work until the pain has completely settled
B. Back strengthening exercises
C. Bed rest until pain improves, then gradual mobilisation
D. Continue usual activity
E. Self referral for physiotherapy

A

D. Continue usual activity

How well did you know this?
1
Not at all
2
3
4
5
Perfectly
104
Q

A24yearoldmandevelopslowbackpainthedayafterfallingwhileplaying tennis. He is usually well and takes no regular medication. He is a laboratory technician.
Which is the most appropriate advice?
A. Avoid work until the pain has completely settled
B. Back strengthening exercises
C. Bed rest until pain improves, then gradual mobilisation
D. Continue usual activity
E. Self referral for physiotherapy

A

D. Continue usual activity

How well did you know this?
1
Not at all
2
3
4
5
Perfectly
105
Q

A44yearoldwomanhasshootingpaininherleftleg.Shehasanadvanced rectal cancer compressing the sciatic nerve. As part of her end of life care, she is taking morphine sulfate modified release (20 mg twice daily), but requests further medication for the pain.
Which is the most appropriate medication to add?
A. Oral amitriptyline
B. Oral diclofenac
C. Oral tramadol
D. Oral venlafaxine
E. Topical ibuprofen

A

A. Oral amitriptyline

How well did you know this?
1
Not at all
2
3
4
5
Perfectly
106
Q

A67yearoldmanpresentstotheEmergencyDepartmentfeelinggenerally unwell with no specific symptoms. He is being treated with neoadjuvant chemotherapy for bowel cancer. He underwent the third cycle 8 days ago.
His temperature is 38.6°C, pulse rate 97 bpm, BP 132/68 mmHg, respiratory rate 12 breaths per minute and oxygen saturation 95% breathing air. Examination is otherwise unremarkable. Blood tests have been taken, but results are not yet available.
Which is the most appropriate next step in management?
A. Admit to medical receiving unit
B. Await blood results before taking further action
C. Discuss with local oncology team for advice
D. Give intravenous broad-spectrum antibiotics
E. Give intravenous fluid therapy

A

D. Give intravenous broad-spectrum antibiotics

How well did you know this?
1
Not at all
2
3
4
5
Perfectly
107
Q

A88yearoldwomanhas30hoursofsevereabdominalpain.Shewas discharged from hospital 4 weeks ago following an aortobifemoral bypass graft.
She has diffuse tenderness of the abdomen with absent bowel sounds. Plain X- ray of the abdomen is unremarkable.
Investigations: Haemoglobin White cell count Urea
114 g/L 18 × 109/L
16.4 mmol/L 158 μmol/L 110 mg/L
(135-180) (3.8–10.0) (2.5–7.8)
(60–120) (<5)
(7.35–7.45) (11–15)
(4.6–6.4) (22–30)
(1–2)
Creatinine
CRP
Arterial blood gas breathing air
pH
PO2
PCO2 Bicarbonate Lactate
A. Acute diverticulitis
B. Acute pancreatitis
C. Incarcerated paraumbilical hernia
D. Mesenteric ischaemia
E. Ruptured aortic aneurysm

A

D. Mesenteric ischaemia

How well did you know this?
1
Not at all
2
3
4
5
Perfectly
108
Q

A67yearoldwomanhasright-sidedpleuriticchestpainandbreathlessnessof sudden onset. She had a bleeding peptic ulcer secondary to NSAID use 4 weeks ago requiring a 2-unit blood transfusion. She has a history of osteoarthritis. She is taking lansoprazole and co-codamol.
Her pulse rate is 112 bpm, BP 114/74 mmHg, respiratory rate 26 breaths per minute and oxygen saturation 94% breathing 40% oxygen. Her chest is clear.
Investigations:
Haemoglobin 93 g/L (115–150) Creatinine 81 μmol/L (60–120)
CT pulmonary angiogram: thrombus in both pulmonary arteries Which is the most appropriate initial treatment?
A. Insertion of vena cava filter
B. Intravenous alteplase
C. Intravenous heparin
D. Oral apixaban
E. Subcutaneous dalteparin sodium

A

C. Intravenous heparin

How well did you know this?
1
Not at all
2
3
4
5
Perfectly
109
Q

A59yearoldwomanhas4daysofsharpcentralchestpainthatradiatesintoher back and is worse on lying flat.
Her temperature is 37.7°C, pulse rate 105 bpm, BP 100/72 mmHg, respiratory rate 18 breaths per minute and oxygen saturation 96% breathing air. Heart sounds are normal, brachial pulses in both arms are synchronous.
Investigations: Haemoglobin White cell count Platelets Neutrophils
D dimers Troponin T
CRP
ECG: see image
152 g/L 11 × 109/L
390 × 109/L 7.5 × 109/L 0.5 mg/L 0.5 μg/L
30 mg/L
(130–175) (3.8–10.0) (150–400) (2.0–7.5)
(<0.5) (<0.01) (<5)
Which is the most likely diagnosis?
A. Acute coronary syndrome
B. Aortic dissection
C. Musculoskeletal chest pain
D. Pericarditis
E. Pneumonia

A

D. Pericarditis

How well did you know this?
1
Not at all
2
3
4
5
Perfectly
110
Q

A 63 year old woman has episodes of irregular palpitations, lasting several days and occurring once a month. She has a history of ischaemic heart disease and type 2 diabetes.
Her ECG confirms atrial fibrillation. The patient wants to discuss the risks before starting anticoagulation (see image).
Which is her lifetime risk of having a stroke related to her atrial fibrillation?
A. 3%
B. 15%
C. 20%
D. 30%
E. 60%

A

E. 60%

How well did you know this?
1
Not at all
2
3
4
5
Perfectly
111
Q

The surgical registrar is running late due to a fault with their car. She asks the surgical Foundation Year 1 doctor (FY1) to obtain written consent from the patients on the afternoon list to avoid a delayed start and possible cancellations. The list is three dupuytren’s contracture releases. The FY1 has not assisted with this operation on this rotation.
Which is the most appropriate action for the FY1 to take?
A. Ask the consultant to consent the patients
B. Consent all of the patients
C. Consent all of the patients and ask the registrar to countersign the forms
D. Only consent the patients without significant medical conditions
E. Refuse to consent the patients

A

A. Ask the consultant to consent the patients

How well did you know this?
1
Not at all
2
3
4
5
Perfectly
112
Q

A 33 year old man has 12 hours of severe, constant pain of sudden onset radiating from the right flank to the groin. He has no significant past medical history.
Urinalysis shows blood 2+, protein negative and leucocytes 1+. Which is the most appropriate initial investigation?
A. Contrast CT of abdomen and pelvis
B. Intravenous urography
C. Non-contrast CT of renal tract
D. Plain X-ray of renal tract
E. Ultrasonography of renal tract

A

C. Non-contrast CT of renal tract

How well did you know this?
1
Not at all
2
3
4
5
Perfectly
113
Q

A 63 year old man presents to his GP for review following addition of chlortalidone to maximal-dose ramipril for BP control. He also has type 2 diabetes mellitus and chronic kidney disease and takes metformin. His creatinine 1 month ago was 115 μmol/L (60–120).
His BP is 133/85 mmHg. Investigations:
Sodium Potassium Urea Creatinine eGFR
135 mmol/L 4.6 mmol/L 9.0 mmol/L
150 μmol/L
44 mL/min/1.73 m2
(135–146) (3.5–5.3) (2.5–7.8)
(60–120) (>60)
Which is the most appropriate management?
A. Stop metformin
B. Repeat urea and electrolytes in 2 weeks
C. Stop chlortalidone
D. Stop ramipril
E. Switch chlortalidone to amlodipine

A

B. Repeat urea and electrolytes in 2 weeks

How well did you know this?
1
Not at all
2
3
4
5
Perfectly
114
Q

A 40 year old woman has 1 day of a painful, swollen left elbow and fever. She has a history of rheumatoid arthritis and takes methotrexate and infliximab.
Her temperature is 38.2°C, pulse rate 100 bpm and BP 119/83 mmHg. The left elbow is swollen and erythematous.
Investigations: White cell count Urea
Creatinine
CRP
16.4 × 109/L (3.8–10.0)
6.7 mmol/L 98 μmol/L 171 mg/L
(2.5–7.8) (60–120)
(<5)
Joint aspiration: no organisms on Gram stain, white cell count 2043/μL (<200), mostly neutrophils, no crystals.
She is advised to take oral paracetamol.
Which is the most appropriate additional management?
A. Inject methylprednisolone into the joint
B. No further treatment pending culture results
C. Start intravenous flucloxacillin
D. Start oral colchicine
E. Start oral prednisolone

A

C. Start intravenous flucloxacillin

How well did you know this?
1
Not at all
2
3
4
5
Perfectly
115
Q

A 33 year old man is found collapsed on the medical ward. He was admitted 3 days ago with urosepsis and is being treated with intravenous antibiotics. He has a history of type 1 diabetes and has been taking his usual doses of subcutaneous insulin.
He is unrousable and is clammy. His capillary blood glucose is 2.1 mmol/L. Which is the most appropriate immediate treatment?
A. 20 mL of 50% glucose by slow intravenous injection
B. 75 mL of 20% glucose by intravenous infusion
C. 150 mL of 5% glucose by intravenous infusion
D. Glucagon 1 mg by intramuscular injection
E. Glucose gel 25 g (contains 10 g glucose) applied to buccal mucosa

A

B. 75 mL of 20% glucose by intravenous infusion

How well did you know this?
1
Not at all
2
3
4
5
Perfectly
116
Q

A 75 year old woman with type 2 diabetes mellitus attends the clinic for review. Her metformin treatment was stopped during a recent hospital admission with a hip fracture, in view of worsening chronic kidney disease [eGFR 28 mL/min/1.73 m2(>60)]. She has a history of osteoarthritis. She is currently taking the maximum dose of gliclazide.
Investigations:
Glycated haemoglobin 79 mmol/mol (20–42)
She is keen to avoid giving herself injections. Which is the most appropriate additional treatment?
A. Acarbose (α-glucosidase inhibitor)
B. Dulaglutide (GLP-1 agonist)
C. Empagliflozin (SGLT2 inhibitor)
D. Pioglitazone (thiazolidinedione)
E. Sitagliptin (DPP4 inhibitor)

A

E. Sitagliptin (DPP4 inhibitor)

How well did you know this?
1
Not at all
2
3
4
5
Perfectly
117
Q

A 62 year old Indian woman visits her GP for an NHS health check. Her BMI is 34 kg/m2. Her BP is 110/84 mmHg.
Urinalysis is normal.
Investigations:
Glycated haemoglobin 45 mmol/mol (20–42) Which is the most appropriate next step in management?
A. Advise a low sugar diet
B. Prescribe metformin
C. Prescribe ramipril
D. Refer her to the Diabetes Prevention Programme
E. Send off a fasting plasma glucose

A

D. Refer her to the Diabetes Prevention Programme

How well did you know this?
1
Not at all
2
3
4
5
Perfectly
118
Q

A 53 year old woman has 6 days of worsening abdominal pain. She has also had recent constipation . There is no rectal bleeding.
Her temperature is 37.8°C, pulse rate 105 bpm and BP 140/85 mmHg. She has tenderness in the left iliac fossa with some guarding . Bowel sounds are normal. Rectal examination shows hard stools only.
Which is the most likely diagnosis?
A. Ischaemic colitis
B. Meckel’s diverticulitis
C. Proctocolitis
D. Rectal carcinoma
E. Sigmoid diverticulitis

A

E. Sigmoid diverticulitis

How well did you know this?
1
Not at all
2
3
4
5
Perfectly
119
Q

A 76 year old man has nausea, fever and rigors. He has foul smelling urine . He was discharged 3 days ago after being treated for a PE. He has been having low molecular weight heparin injections twice daily since the diagnosis. He had a cholecystectomy 12 years ago.
His temperature is 39.7°C, pulse rate 100 bpm and BP 92/41 mmHg.
Investigations:
APTT 43 seconds (22–41) PT 18 seconds (10–12)
Which is the most likely cause of the prolonged prothrombin time?
A. Disseminated intravascular coagulation
B. Liver disease
C. Low molecular weight heparin
D. Lupus anticoagulant
E. Vitamin K deficiency

A

A. Disseminated intravascular coagulation

How well did you know this?
1
Not at all
2
3
4
5
Perfectly
120
Q

A 62 year old man has a right-sided hearing loss. There has been slow deterioration over the previous two years, and he is now also troubled by non- pulsatile, right-sided tinnitus that prevents him from sleeping.
A pure-tone audiogram shows a right-sided high-frequency hearing loss. He has normal tympanometry bilaterally.
Which is the most appropriate diagnostic investigation?
A. Cerebral angiography
B. CT of head
C. CT of petrous temporal bones
D. MR imaging of internal acoustic meatus
E. PET–CT of brain

A

D. MR imaging of internal acoustic meatus

121
Q

A 65 year old woman has had bloody diarrhoea six times each day for the past 4 weeks and is feeling unwell. She has noticed urgency to pass stool and has to get up in the night to pass stool. Her left eye has been red, but not painful.
Her temperature is 38.5°C, pulse rate 109 bpm, BP 110/70 mmHg and respiratory rate 22 breaths per minute.
Investigations:
Stool culture: negative
Which is the most likely diagnosis?
A. Adenocarcinoma of the colon
B. Crohn’s disease
C. Irritable bowel syndrome
D. Microscopic colitis
E. Ulcerative colitis

A

E. Ulcerative colitis

122
Q

A 28 year old man presents to his GP with 7 weeks of right iliac fossa pain, weight loss of 3 kg and diarrhoea five times a day. He previously opened his bowels once daily. He has not travelled abroad. He lives with his partner, who is well.
Rectal examination is normal.
Investigations:
Haemoglobin
Albumin
White cell count
Platelets
Erythrocyte sedimentation rate ALT
ALP Bilirubin
129 g/L (Men: 135-180 g/l Women: 115-160 g/l)
Which is the most appropriate next investigation?
A. Antimitochondrial antibodies
B. Anti-tissue transglutaminase antibodies
C. Faecal calprotectin
D. Faecal occult blood testing
E. Stool cultures

A

E. Stool cultures

123
Q

A 49 year old woman is undergoing investigation for headaches. She has no visual symptoms.
Her visual acuity is 6/5 bilaterally. There is a visual field defect.
Investigations:
MR scan of brain: pituitary adenoma indenting the upper optic chiasm
Which is the most likely visual field defect?
A. Bilateral concentric visual loss
B. Bilaterally enlarged blind spots
C. Bitemporal hemianopia
D. Left-sided homonymous hemianopia
E. Right-sided homonymous hemianopia

A

C. Bitemporal hemianopia

124
Q

A 36 year old man has recurrent episodes of collapse over 4 months. When laughing, his muscles feel limp and he falls to the floor, but remains conscious. He has a history of anxiety and depression and takes citalopram. His father died from an ischaemic stroke aged 59 years.
Which is the most likely diagnosis?
A. Cardiogenic syncope
B. Cataplexy
C. Cough syncope
D. Dissociative (non-epileptic) seizures
E. Epilepsy

A

B. Cataplexy

125
Q

A 58 year old woman has 6 months of worsening right-sided hip pain caused by osteoarthritis. Her pain is partially controlled by regular paracetamol and ibuprofen gel. She remains active. She has peptic ulcer disease and heart failure. Her medication includes bisprolol fumarate, aspirin and simvastatin.
She has crepitus on active and passive movements of both hips. Her BMI is 30 kg/m2.
Which is the most appropriate management?
A. Add celecoxib
B. Add oral ibuprofen and lansoprazole
C. Refer for acupuncture
D. Refer for knee replacement surgery
E. Switch paracetamol to regular co-codamol

A

E. Switch paracetamol to regular co-codamol

126
Q

A 37 year old man has 4 weeks of pain on the radial side of his left wrist. The pain is worse when he plays the piano. He has no history of trauma.
He has swelling and tenderness over the distal end of the left radial styloid. The pain is exacerbated by ulnar deviation of the wrist when the thumb is clasped in the palm.
Which is the most likely diagnosis?
A. de Quervain tendinopathy
B. Ganglion cyst
C. Intersection syndrome
D. Osteoarthritis of the trapeziometacarpal joint
E. Scaphoid fracture

A

A. de Quervain tendinopathy

127
Q

A 41 year old woman is admitted following a significant overdose of amitriptyline and propranolol taken 30 minutes ago.
Her GCS is 15. Her pulse rate is 80 bpm, BP 134/90 mmHg, respiratory rate 14 breaths per minute and oxygen saturation 98% breathing air.
Investigations:
Arterial blood gas breathing air
pH
PO2
PCO2 Bicarbonate
7.42 11.7 kPa
4.9 kPa 24 mmol/L
(7.35–7.45) (11–15)
(4.6–6.4) (22–30)
ECG: sinus rhythm, QRS duration 100 ms (80–120) Which is the most appropriate next step in management?
A. Arrange gastric lavage
B. Give oral activated charcoal
C. Give oral ipecacuanha
D. Start IV 1.26% sodium bicarbonate
E. Start IV acetylcysteine

A

B. Give oral activated charcoal

128
Q

A 48 year old man has 8 months of a painful, stiff left shoulder. The pain is dull and worse at night. There is no history of trauma. He has type 1 diabetes mellitus.
He has reduced passive and active range of movement in the left shoulder with pain at extremes of range of motion. Neck movements are normal. Power and sensation are normal.
Investigations: CRP 6
Which is the most likely diagnosis?
A. Adhesive capsulitis
B. Glenohumeral osteoarthritis
C. Polymyalgia rheumatica
D. Rotator cuff tear
E. Subacromial bursitis

A

A. Adhesive capsulitis

129
Q

A 57 year old woman is reviewed following a bone densitometry scan (DXA). She was recently found to have polymyalgia rheumatica, and treatment with oral prednisolone was started. The T-score for the bone mineral density in her femoral neck is –3.9. She is advised to take colecalciferol with calcium carbonate.
Which is the most appropriate additional treatment?
A. Alendronic acid
B. Denosumab
C. No additional treatment
D. Raloxifene hydrochloride
E. Teriparatide

A

A. Alendronic acid

130
Q

A 66 year old woman presents to the GP with 7 weeks of intermittent dizziness that she describes as ‘the room spinning’. The episodes start suddenly, are usually triggered by looking upwards, and last less than a minute before resolving completely. She reports nausea during these episodes. She is otherwise well.
Otoscopic and neurological examinations are normal. Which is the most likely diagnosis?
A. Acoustic neuroma
B. Benign paroxysmal positional vertigo
C. Cerebellar lesion
D. Ménière’s disease
E. Vestibular neuritis

A

B. Benign paroxysmal positional vertigo

131
Q

A 62 year old man becomes increasingly breathless 3 hours after the removal of a chest drain. This was inserted to drain a right-sided pleural effusion secondary to lymphoma . Three litres of bloodstained fluid were drained before removal.
Which is the most likely diagnosis?
A. Aspiration pneumonia
B. Pneumothorax
C. Pulmonary embolism
D. Pulmonary oedema
E. Recurrence of effusion

A

B. Pneumothorax

132
Q

A 38 year old woman has abdominal pain 7 days after a laparoscopic sleeve gastrectomy for weight loss.
Her temperature is 36.5°C, pulse rate 110 bpm, BP 120/72 mmHg and oxygen saturation 96% breathing oxygen 4 L/min via nasal prongs. She has reduced breath sounds in both bases. She has abdominal tenderness with guarding and reduced bowel sounds.
Which is the most appropriate diagnostic investigation?
A. Abdominal X-ray
B. Barium enema
C. CT of abdomen
D. Erect chest X-ray
E. Point-of-care FAST scan

A

C. CT of abdomen

133
Q

A 25 year old woman reports feeling short of breath 12 hours after abdominal surgery for adhesions. She has a large volume of blood in her abdominal drains. Her pulse rate is 119 bpm, BP 92/54 mmHg, respiratory rate 24 breaths per minute and oxygen saturation 96% breathing oxygen 4 L/minute via nasal cannulae.
She is treated with an IV fluid bolus, and the surgical registrar is informed. Which is the most appropriate next management step?
A. Activate major haemorrhage protocol
B. Give IV noradrenaline/norepinephrine
C. Give IV tranexamic acid
D. Give prothrombin complex concentrate
E. Increase oxygen to 15 L/min via a non-rebreather mask

A

A. Activate major haemorrhage protocol

134
Q

A 59 year old man presents to the GP for review. He has type 1 diabetes mellitus, hypertension, ischaemic heart disease and CKD stage 4 [recent eGFR 25 mL/min/1.73 m2(> 60)]. He is taking insulin, lisinopril, metoprolol tartrate and aspirin.
His BP is 160/96 mmHg. He has pitting oedema to the mid shin. Investigations:
Sodium Potassium Urea Creatinine eGFR
144 mmol/L 5.4 mmol/L 19.2 mmol/L
237 μmol/L
26 mL/min/1.73 m2
(135–146) (3.5–5.3) (2.5–7.8) (60–120) (>60)
Urinalysis: protein 2+
Which is the most appropriate therapeutic change?
A. Add indapamide
B. Add furosemide
C. Add spironolactone
D. Stop lisinopril
E. Switch lisinopril to losartan

A

B. Add furosemide

135
Q

A 30 year old woman is brought to the resuscitation room of the Emergency Department after after being kicked by a horse . She is 28 weeks pregnant. Her cervical spine is immobilised.
She is conscious and asks about her baby. Her temperature is 37°C, pulse rate 100 bpm, BP 78/56 mmHg and oxygen saturation 99% breathing 40% oxygen . Fluid resuscitation has been started and bloods have been sent.
Which is the most appropriate next step?
A. Image the long bones and pelvis, and orthopaedic review
B. Perform a full primary survey
C. Scan the baby and arrange fetal monitoring
D. Transfer to theatre for immediate delivery of baby
E. X-ray cervical spine

A

B. Perform a full primary survey

136
Q

A 55 year old man is brought to the Emergency Department with 2 hours of severe left sided chest pain. A pre-hospital ECG shows left bundle branch block, for which paramedics gave aspirin and glyceryl trinitrate spray.
On arrival at hospital, he is sweating and distressed . His pulse rate is 85 bpm, BP 99/54 mmHg, respiratory rate 22 breaths per minute and oxygen saturation 96% breathing air. His GCS score is 15/15.
Which is the most appropriate management?
A. Continue breathing air
B. Start 28% oxygen via a Venturi mask
C. Start 40% oxygen via a Venturi mask
D. Start oxygen 2 L/min via nasal cannulae
E. Start oxygen 15 L/min via a non-rebreathe mask

A

A. Continue breathing air

137
Q

A 25 year old man has taken an overdose of 16 paracetamol tablets. He became very sad earlier that day after an argument with a friend. His mood changes between being happy and sad several times per week. He struggles with concentration and gets angry easily. He has frequently been excessively drunk on nights out with friends. He has never experienced psychotic symptoms.
Which is the most likely diagnosis?
A. Adjustment disorder
B. Bipolar disorder
C. Borderline personality disorder
D. Dissocial personality disorder
E. Unipolar depressive episode

A

C. Borderline personality disorder

138
Q

A 52 year old woman has a brief episode of dizziness on standing . She has had 4 days of dysuria, loin pain and fever. She has been feeling tired for 4 months and has lost 3 kg in weight.
Her temperature is 37.4°C, pulse rate 90 bpm, BP 100/55 mmHg lying and 90/50 mmHg sitting, respiratory rate 18 breaths per minute and oxygen saturation 95% breathing air. Her JVP is not visible.
Investigations: Haemoglobin White cell count Platelets
Sodium Potassium Urea Creatinine
Random plasma glucose
106 g/L 14 × 109/L
201 × 109/L
130 mmol/L 5.6 mmol/L 9.5 mmol/L
98 μmol/L 3.2 mmol/L
(115–150) (3.8–10.0) (150–400)
(135–146) (3.5–5.3) (2.5–7.8) (60–120)
12-lead ECG
Which is the most appropriate additional investigation?
A. CT of head
B. CT pulmonary angiography
C. Echocardiography
D. Plasma cortisol and adrenocorticotropic hormone
E. Urinary and serum osmolality

A

D. Plasma cortisol and adrenocorticotropic hormone

139
Q

A 39 year old woman with metastatic lung cancer has 24 hours of back pain at the L1 level. She has also developed leg weakness and urinary retention.
Which is the most appropriate investigation?
A. CT of spine
B. Isotope bone scan
C. MR imaging of spine
D. PET–CT
E. X-ray of thoracic and lumbar spine

A

C. MR imaging of spine

140
Q

A 75 year old woman attends her GP surgery with breathlessness on exertion and a cough productive of white sputum throughout the day. She has never had haemoptysis and has lost 2kg in weight . She describes two chest infections in the past year treated with a short course of steroids and antibiotics. She has a 15 pack-year smoking history.
Her heart rate is 82. Chest X-ray is normal.
Which is the most appropriate investigation to establish the diagnosis?
A. Cardiopulmonary exercise test
B. Echocardiography
C. High resolution CT scan of thorax
D. Spirometry
E. Sputum cytology

A

D. Spirometry

141
Q

A 32 year old man presents to the Emergency Department with 60 minutes of palpitations that started during a night out dancing with friends. This is associated with chest pain and shortness of breath.
Investigations: ECG (see image)
What diagnosis is shown on the ECG?
A. Atrial fibrillation
B. First degree heart block
C. Sinus tachycardia
D. Supraventricular tachycardia E. Ventricular tachycardia

A

D. Supraventricular tachycardia

142
Q

A 82 year old woman is oliguric 8 hours after an emergency sigmoid colectomy for a perforated diverticulum. She has a history of chronic kidney disease stage 3, peripheral vascular disease and type 2 diabetes mellitus. She takes metformin and ramipril.
Her pulse rate is 110 bpm, BP 94/60 mmHg and JVP not visible. Her urine output has been 10 mL per hour for the past 4 hours. She has been given 2 L of hartmann’s solution since returning from theatre.
Investigations:
Urea 14.9 mmol/L (2.5–7.8) Creatinine 188 μmol/L (60–120)
Which is the most likely cause of her oliguria?
A. Acute tubular necrosis
B. Catheter obstruction
C. Intraoperative ureteric injury
D. Prerenal hypovolaemia
E. Renal artery occlusion

A

D. Prerenal hypovolaemia

143
Q

A 39 year old man is brought to the Emergency Department having hit his head on a tree trunk when he lost control of his bicycle.
He has a minor scalp abrasion. His GCS is 15/15 and there are no focal neurological signs.
Seven hours later he collapses, at which point his GCS is 6/15. A CT scan of brain reveals a lens-shaped haemorrhage in the right parietal region.
Which is the most likely vessel to be injured?
A. Anterior communicating artery
B. Basilar artery
C. Bridging veins
D. Middle meningeal artery
E. Vertebral artery

A

D. Middle meningeal artery

144
Q

A 68 year old woman has worsening discomfort in the chest over 11 days. She gives a 2 month history of loosing 3kg in weight. She has a dry cough and is a non-smoker. She retired as a builder 15 years ago.
She has a temperature of 37.1°C. Both lung fields sound clear. Her chest X-ray is shown (see image).
Which is the most likely underlying diagnosis?
A. Mesothelioma
B. Metastatic cancer
C. Pulmonary fibrosis
D. Pulmonary tuberculosis
E. Sarcoidosis

A

B. Metastatic cancer

145
Q

Researchers compare the diagnostic accuracy of a new serological test for SARS-Cov-2 against the nose and throat swab currently being used for clinical diagnosis in the UK. The new test detects all 23 confirmed cases of the disease that were positive by the current test. 92% (230/250) of patients who tested negative with the current test are also negative by the new rapid diagnostic test.
Which statement is correct?
A. The false negative rate of the new test is 0%
B. The false positive rate of the new test is 92%
C. The sensitivity of the new test is 92%
D. The specificity of the new test is 8%
E. The specificity of the new test is 100%

A

A. The false negative rate of the new test is 0%

146
Q

A 24 year old man has 12 hours of a severe headache of sudden onset. When the headache started, he lost consciousness briefly and vomited; he also reports photophobia . He has a past history of severe hypertension .
Neurological examination is normal. A non-contrast CT scan of brain is normal. What is the most appropriate next step in management?
A. Carotid Doppler studies
B. Lumbar puncture
C. MR scan of brain
D. MR imaging of brain with MR venography E. No future investigations required

A

B. Lumbar puncture

147
Q

A 33 year old man has 6 months of an itchy, well demarcated rash that has partially responded to topical moisturiser.
There are lesions symmetrically distributed over the extensor surface of his elbows, scalp and back, which are 2–10 cm in diameter (see image).
Which is the most likely diagnosis?
A. Atopic dermatitis
B. Pityriasis versicolor
C. Plaque psoriasis
D. Seborrhoeic dermatitis
E. Tinea corporis infection

A

C. Plaque psoriasis

148
Q

A 21 year old woman has recurrent pulsating frontal headaches lasting up to 2 days associated with nausea. The headache is usually preceded by numbness that starts in the hand, travels up the arm and then involves the face, lips and tongue. These headaches cause her to take time off work up to four times each month.
Which is the most appropriate treatment to reduce the frequency of these episodes?
A. Acupuncture
B. Gabapentin
C. Propranolol
D. Sumatriptan
E. Venlafaxine

A

C. Propranolol

149
Q

A 60 year old man reports increasing fatigue. He has noticed intermittent difficulty swallowing, usually in the evening. He is a smoker. His only regular medication is levothyroxine.
There is bilateral mild ptosis. The rest of the neurological examination is normal. Which is the most likely diagnosis?
A. Guillain–Barré syndrome
B. Migraine
C. Multiple sclerosis
D. Myasthenia gravis
E. Stroke

A

D. Myasthenia gravis

150
Q

Six students living in the same hall of residence have developed diarrhoea and vomiting within 48 hours of each other. There is no blood in the stool, they have not eaten similar foods and have no history of foreign travel.
A doctor requests stool samples from each to confirm the likely diagnosis. Which test is most likely to confirm the diagnosis?
A. Clostridium difficile enzyme assay and toxin detection
B. Faecal elastase test
C. Stool for culture and sensitivity
D. Stool for viral PCR
E. Stool microscopy for ova, cysts and parasites

A

D. Stool for viral PCR

151
Q

An 88 year old woman has recurrent falls. She says that she cannot feel where her feet are.
There is reduced pinprick sensation up to the level of her mid calf and proprioception is impaired. Knee reflexes are brisk, but ankle reflexes are absent.
Investigations: Haemoglobin White cell count Platelets
MCV
Which is the most likely deficiency?
A. Folate
B. Iron
C. Pyridoxine
D. Thiamine
E. Vitamin B 12

A

E. Vitamin B 12

152
Q

A 55 year old man has had lower back pain and fatigue for 3 months.
Investigations:
Haemoglobin
White cell count
Platelets
Corrected calcium 2.90 mmol/L
(130–175) (3.8–10.0) (150–400) (2.2–2.6)
110 g/L 5.8 × 109/L
120 × 109/L
Albumin 29 g/L (35–50)
Serum electrophoresis: monoclonal Ig kappa peak
Bone marrow biopsy: foci of plasma cells, which account for 18% of all haematopoietic cells
Which is the most likely diagnosis?
A. Acute lymphoblastic lymphoma
B. Chronic lymphocytic leukaemia
C. Extramedullary plasmacytoma
D. Multiple myeloma
E. Waldenström’s macroglobulinaemia

A

D. Multiple myeloma

153
Q

A 54 year old woman has reduced urine output 24 hours after admission with right lower lobe consolidation due to community-acquired pneumonia. She has been treated with intravenous amoxicillin and clarithromycin, but remains breathless. Her creatinine was 82 μmol/L (60–120) on admission.
Her temperature is 38.0°C, pulse rate 106 bpm, BP 102/50 mmHg and oxygen saturation 95% breathing 4 L/min oxygen via nasal prongs. Her urine output is 250 mL over the past 12 hours. Her urinalysis has protein 1+.
Investigations: Haemoglobin White cell count Platelets
Urea Creatinine
119 g/L 16.9 × 109/L
95 × 109/L
15.5 mmol/L 160 μmol/L
(115–150) (3.8–10.0) (150–400)
(2.5–7.8) (60–120)
Which is the most likely cause of her acute kidney injury?
A. Drug-induced interstitial nephritis
B. Haemolytic uraemic syndrome
C. Infection-related glomerulonephritis
D. Renal hypoperfusion
E. Systemic vasculitis

A

D. Renal hypoperfusion

154
Q

A 70 year old woman has an ulcer above the left medial malleolus. She has a history of type 2 diabetes mellitus. She smokes 10 cigarettes per day.
The ulcer is 10 × 5 cm and superficial. She has brown discolouration of both lower legs. The skin has a thickened, waxy feel. Her BMI is 34.

Which is the most likely type of ulcer in this patient?
A. Arterial
B. Inflammatory C. Malignant
D. Neuropathic E. Venous

A

E. Venous

155
Q

A 67 year old man has 3 weeks of progressive ankle oedema. He has a history of hypertension, treated with amlodipine. He is a lifelong heavy smoker and drinks 12 units of alcohol per week.
His BP is 125/85 mmHg and oxygen saturation 98% breathing air. His JVP is 4 cm above the sternal angle. He has marked bilateral pitting ankle oedema. He has dull percussion note at both bases with reduced breath sounds.
Investigations: Creatinine
Fasting glucose Total cholesterol
Albumin
Urinary protein:creatinine ratio Urine microscopy
Which is the most likely diagnosis?
85 μmol/L
5.7 mmol/L 9 mmol/L
15 g/L
568 mg/mmol
(60–120)
(3.0–6.0) (<5.0)
(35–50) (<30)
no cells, no casts
A. Cardiac failure
B. Nephritic syndrome
C. Nephrotic syndrome
D. Rapidly progressive glomerulonephritis
E. Renovascular disease

A

C. Nephrotic syndrome

156
Q

An 81 year old woman sustains a fractured neck of femur on falling while walking on carpet in her flat. She recovers well after surgery and rehabilitation, and is discharged home. She is advised to take calcium and vitamin D supplements.
Which additional medication should be prescribed to reduce the risk of a future fracture?

A. Alendronic acid
B. Denosumab
C. Raloxifene hydrochloride D. Teriparatide
E. Zoledronic acid

A

E. Zoledronic acid

157
Q

A 68 year old man has eight weeks of back pain. It sometimes wakes him at night, and he is feeling increasingly tired. He has no history of back problems and has no history of recent trauma.
He has tenderness over L3 and L4 vertebrae.
Investigations:
Haemoglobin 137 g/L (130–175)Erythrocyte sedimentation rate 55 mm/hr (< 20)
Creatinine 72 μmol/L (60–120)
Calcium 2.5 mmol/L (2.2–2.6)Serum protein electrophoresis: no paraprotein
Which is the most appropriate next investigation?
A. CT scan abdomen and pelvis
B. DEXA scan
C. HLA-B27 antigen
D. Isotope bone scan
E. X-ray lumbar spine

A

E. X-ray lumbar spine

158
Q

A 17 year old girl has a florid skin rash for 24 hours. She has been feeling unwell for the past week with intermittent abdominal pain.
She has a palpable, purpuric rash on her lower limbs and buttocks. Her temperature is 37.2°C, pulse rate 70 bpm and BP 122/80 mmHg. Her urinalysis has blood 3+, protein 2+, nitrites negative, leucocytes negative
Investigations: Haemoglobin 109/L Platelets
Urea Creatinine
122 g/L (3.8–10.0)
320 × 109/L
3.2 mmol/L 60 μmol/L
(115–150)White cell count (150–400)
(2.5–7.8) (60–120)
8.9 ×

Which is the most likely diagnosis?
A. IgA vasculitis (Henoch–Schönlein purpura) B. Meningococcal septicaemia
C. Microscopic polyangiitis
D. Postinfectious glomerulonephritis
E. Systemic lupus erythematosus

A

A. IgA vasculitis (Henoch–Schönlein purpura)

159
Q

A 67 year old woman becomes unwell whilst attending the diabetes foot clinic. She is anxious and noticeably tremulous as she drinks from her water bottle.
She is sweaty but apyrexial. Her pulse rate is 98 bpm, BP 128/76 mmHg and oxygen saturation 96% breathing air. Her capillary blood glucose is 2.1 mmol/L.
Which is the most appropriate next step in management?
A. Intramuscular glucagon
B. Intravenous 10% glucose
C. Intravenous 20% glucose
D. Oral glucose gel (GlucoGel ® )
E. Oral glucose tablets

A

E. Oral glucose tablets

160
Q

An 82 year old woman has constipation and passes infrequent, hard stools. She has hypertension, overactive bladder symptoms and type 2 diabetes mellitus. She takes amlodipine, doxazosin, gliclazide, metformin and oxybutynin.
Which medication is most likely to be worsening her constipation?
A. Amlodipine
B. Doxazosin
C. Gliclazide
D. Metformin
E. Oxybutynin

A

E. Oxybutynin

161
Q

A 65 year old man reports sudden onset of visual disturbance with flashing lights, floaters and loss of vision in the upper outer quadrant of his right eye. He has a history of hypertension.
Which is the most likely diagnosis?
A. Acute glaucoma
B. Central retinal artery occlusion
C. Central retinal vein occlusion
D. Retinal detachment
E. Vitreous haemorrhage

A

D. Retinal detachment

162
Q

A 70 year old woman develops pyrexia and reduced oxygen saturation 2 days after an elective subtotal gastrectomy. Her postoperative pain control has been difficult, which has limited her ability to have chest physiotherapy and to mobilise.
Her temperature is 37.8°C, pulse 84 bpm and oxygen saturation 92% breathing 35% oxygen. Her BMI 36. There is reduced breath sounds at both lung bases. Her abdomen is soft, with tenderness around her wound. Her drain has serous output.
Which is the most likely postoperative complication?
A. Anastomotic leak
B. Atelectasis
C. Pneumothorax
D. Pulmonary embolism
E. Wound infection

A

B. Atelectasis

163
Q

A 69 year old man has 6 months of back pain extending into his buttocks and back of his thighs. The pain is worse on standing and on walking, and is relieved when he sits down and leans forward. He has diet-controlled type 2 diabetes mellitus. He is an ex-smoker with a 40 pack-year history.
His BP is 178/95 mmHg. He has weakness of hip flexion bilaterally. His peripheral pulses are palpable.
Which is the most likely diagnosis?
A. Diabetic amyotrophy
B. Lumbar disc prolapse
C. Lumbar spinal stenosis
D. Osteoarthritis of hips
E. Peripheral arterial disease

A

C. Lumbar spinal stenosis

164
Q

A 62 year old man presents to the Emergency Department following a road traffic collision. He has severe bruising of the right upper shoulder from the seat belt, but no other injuries.
Chest X-ray to exclude a pneumothorax shows a 2 cm mass in the right upper zone.
Which is the most appropriate next investigation?
A. CT scan of chest
B. Lateral chest X-ray
C. MR scan of chest
D. PET scan of chest
E. Technetium bone scan

A

A. CT scan of chest

165
Q

A 28 year old woman returns to the dermatology clinic 2 weeks after surgery to excise a 1.1 cm pigmented lesion on her right lower leg.
The histopathological report states that the lesion is a superficial spreading melanoma with a Clark level 3, Breslow depth 0.9 mm, mitotic index of 1/mm2and no ulceration. It has been completely excised.
Which is the most important pathological prognostic indicator?
A. Breslow depth
B. Clark level
C. Diameter of lesion
D. Melanoma subtype
E. Mitotic index

A

A. Breslow depth

166
Q

A 42 year old woman has 12 hours of colicky central abdominal pain, vomiting, abdominal distension and increased bowel sounds. She had a ruptured appendix aged 20 years.
Plain abdominal X-ray: see image.
<please insert picture MSCAA-260416-16863_figure_1.jpg. In your exported file, save and unpack the folder with images to be able to insert the image above this caption.>
Which is the most appropriate initial management?
A. Flatus tube insertion
B. Intravenous antibiotics
C. Laparoscopy
D. Laparotomy
E. Nasogastric tube insertion

A

E. Nasogastric tube insertion

167
Q

A 59 year old woman has loin pain and dysuria.
Her temperature is 39°C, pulse rate 108 bpm, BP 90/60 mmHg and
respiratory rate 18 breaths per minute.
Investigations:
Haemoglobin 130 g/L White cell count 22.0 × 109/L
40 × 109/L 20 seconds 60 seconds
1.0 g/dL
Which is the most likely explanation for her thrombocytopenia?
A. Disseminated intravascular coagulation
B. Haemophilia B
C. Idiopathic thrombocytopenic purpura
D. Vitamin B 12 deficiency
E. von Willebrand disease

A

A. Disseminated intravascular coagulation

168
Q

A previously well 22 year old student is brought to the Emergency Department by his flatmate having briefly lost consciousness. He explains that he had gone to bed after a party and been woken at 03:00 by the need to pass urine. He remembers feeling faint and nauseated while bearing down to pass urine, lost consciousness and fell to the floor.
Examination and ECG are normal. Which is the most likely diagnosis?
A. Alcohol related seizure
B. Cardiac dysrhythmia
C. Ecstasy toxicity
Platelets PT
APTT Fibrinogen
(150–400) (10–12) (22–41) (1.5–4.0)
(115–150) (3.8–10.0)

D. Hypoglycaemia
E. Vasovagal syncope

A

E. Vasovagal syncope

169
Q

A 19 year old woman requires an urgent appendicectomy. The
anaesthetist explains that it will be necessary for the patient to breathe some oxygen from a face mask before induction of anaesthesia, and that she will feel some pressure on the front of her neck as she goes to sleep. The patient asks why.
What is the purpose of cricoid pressure?
A. It facilitates endotracheal intubation
B. It prevents the passage of gastric contents into the airway
C. It reduces the haemodynamic response to endotracheal intubation D. It reduces the risk of vomiting
E. It stabilises the neck in a neutral position

A

B. It prevents the passage of gastric contents into the airway

170
Q

A 45 year old man with pain caused by cancer has been using opioids to control his pain very successfully. He is taking a regular dose of MST Continus® 60 mg 12-hourly orally. He has been using three breakthrough doses (oral morphine 20 mg) per day for the past week.
Which is the most appropriate opioid prescription?
A. Diamorphine 60 mg subcutaneously over 24 h by syringe driver
B. Morphine 90 mg subcutaneously over 24 h by syringe driver
C. MST Continus ® 60 mg 12-hourly and morphine 30 mg as required (up to 4- hourly) orally
D. MST Continus ® 90 mg 12-hourly and morphine 20 mg as required (up to 4- hourly) orally
E. MST Continus ® 90 mg 12-hourly and morphine 30 mg as required (up to 4- hourly) orally

A

E. MST Continus ® 90 mg 12-hourly and morphine 30 mg as required (up to 4- hourly) orally

171
Q

A 25 year old man has left-sided chest pain that is worsened by lying back and when he takes a deep breath. He has had a recent respiratory tract infection.
Investigations: Troponin T 0.025 μg/L (<0.01) ECG: see image
<please insert picture MSCAA-260116-17606_figure_1.jpg. In your exported file, save and unpack the folder with images to be able to insert the image above this caption.>
Which is the most likely diagnosis?
A. Pericardial effusion
B. Pericarditis
C. Pulmonary embolism
D. ST-elevation myocardial infarction
E. Unstable angina

A

B. Pericarditis

172
Q

A 18 year old woman has 6 hours of severe dizziness and nausea. She says that the room is constantly spinning round and she has vomited several times. The dizziness is worse when she opens her eyes. She reports that her hearing has not changed.
She has nystagmus with the fast phase to the left, which does not fatigue. Which is the most likely diagnosis?
A. Benign positional vertigo
B. Cerebellar tumour
C. Ménière’s disease
D. Vestibular migraine
E. Vestibular neuronitis

A

E. Vestibular neuronitis

173
Q

A 60 year old man visits his GP as he wants to start training for a 5 km race for charity. He plans to run 3–5 km three times per week. He describes occasional central chest tightness when he walks up hills. He has a history of type 2 diabetes mellitus and COPD for which he is taking metformin and using an as-required salbutamol inhaler.
His BP is 162/94 mmHg. His BMI is 32.
Which aspect of his clinical background is a contra-indication to his training plan?
A. BP >160/90 mmHg
B. COPD
C. Exertional chest tightness
D. Obesity
E. Type 2 diabetes mellitus

A

C. Exertional chest tightness

174
Q

A 75 year old man visits his GP with one episode of visible haematuria. His temperature is 36.2°C and BP 142/80 mmHg.
Investigations:
Midstream urine: red blood cells and epithelial cells, no microbial growth
Urinalysis: blood 2+, leucocytes negative, protein negative, nitrite negative Which is the most appropriate next step?
A. Arrange an ultrasound scan of renal tract
B. Arrange CT urography
C. Check serum prostate specific antigen
D. Refer for urology opinion
E. Repeat midstream urine sample for culture and sensitivity

A

D. Refer for urology opinion

175
Q

A 73 year old man has 3 months of increasing weakness of his right hand with reduced sensation of the forearm.
There is wasting of all the intrinsic muscles of the right hand. There is weakness of finger abduction and adduction, and thumb adduction. Finger flexion is normal. There is mild altered light touch sensation along the ulnar aspect of the forearm. The biceps, supinator and triceps reflexes are normal. The lower limbs and the left arm are normal.
Where is the most likely site of the lesion causing his symptoms?
A. Median nerve in the forearm
B. Median nerve in the wrist
C. Spinal cord C8 level
D. T1 nerve root
E. Ulnar nerve at the elbow

A

D. T1 nerve root

176
Q

An 84 year old woman is admitted 24 hours earlier following a fall in her home. She has mild dementia and osteoarthritis. She is taking donepezil and ibuprofen.
Her temperature is 36.2°C, pulse rate 77 bpm and BP 152/88 mmHg. Her abbreviated mini-mental test score is 6/10. She has some bruising on her face.
A midstream urine culture result 24 hours later shows a mixed growth of organisms with no leucocytes.
Which is the most likely explanation for the midstream urine culture result?
A. Bladder carcinoma
B. Colovesical fistula
C. Contaminated urine specimen
D. Cystitis

E. Interstitial nephritis

A

C. Contaminated urine specimen

177
Q

A 70 year old woman has 6 weeks of generalised shoulder and hip pain and stiffness. She feels excessively tired.
Her temperature is 37.5°C. She has reduced range of movement at the shoulders, but no muscle tenderness. There is synovial thickening and tenderness at the right wrist.
Investigations:
Erythrocyte sedimentation rate 85 mm/hr (<20) Creatine kinase 110 U/L (25–175)
Which is the most appropriate treatment?
A. Allopurinol
B. Hydroxychloroquine
C. Ibuprofen
D. Prednisolone
E. Sulfasalazine

A

D. Prednisolone

178
Q

A 53 year old man has increasing abdominal swelling over several weeks. He has developed severe abdominal pain over the past 12 hours. He drinks one to two bottles of vodka per day.
He has jaundice, and has spider naevi and prominent veins on his abdominal wall. His temperature is 37.6°C. His abdomen is diffusely tender.
Investigations: Haemoglobin White cell count Platelets
INR
ALT ALP
136 g/L 9.6 × 109/L
160 × 109/L 1.2
350 IU/L 140 IU/L
(130–175) (3.8–10.0) (150–400)
(1.0)
(10–50) (25–115)
Bilirubin 78 μmol/L (<17)
Ultrasound scan of abdomen shows ascites with mild hepatosplenomegaly. Which is the most appropriate next step?
A. Ascitic tap
B. CT of abdomen
C. Hepatitis serology
D. Percutaneous liver biopsy
E. Ultrasonography of abdomen

A

A. Ascitic tap

179
Q

A 65 year old woman has had a painful, red and watery left eye for 3 days. Visual acuity is 6/24 on the left and 6/12 on the right. Fundoscopy is normal.
Which is the most appropriate next step in management?
A. Arrange ophthalmology outpatient appointment
B. Prescribe chloramphenicol eye drops
C. Prescribe prednisolone eye drops
D. Prescribe sodium cromoglicate eye drops
E. Refer to the ophthalmology department as an emergency

A

E. Refer to the ophthalmology department as an emergency

180
Q

A 75 year old man with metastatic prostate cancer develops lumbar discomfort, weakness in both legs and urinary incontinence.
Power is 2/5 in all lower limb muscle groups, he has bilateral extensor plantar reflexes, and sensation to pinprick is reduced below the umbilicus.
Which is the most appropriate immediate treatment?
A. Dexamethasone
B. Gabapentin
C. Morphine sulfate
D. Spinal decompression
E. Spinal radiotherapy

A

A. Dexamethasone

181
Q

A 45 year old woman notices a painless breast lump while showering. She makes an urgent appointment with her GP because she is worried about breast cancer. She is seen in the breast clinic for triple assessment.
What does triple assessment involve?
A. Clinical examination, breast imaging and bone scan B. Clinical examination, breast imaging and core biopsy C. MR scan, bone scan and excision biopsy
D. MR scan, genetic screening and excision biopsy
E. PET scan, genetic screening and core biopsy

A

B. Clinical examination, breast imaging and core biopsy

182
Q

A 52 year old Afro-Carribbean man is referred to the hypertension clinic with an average daytime reading on ambulatory BP monitoring of 164/105 mmHg. He has no other medical history of note and is currently taking no medication.
In clinic, his BP is 158/98 mmHg. His 10-year cardiovascular risk is calculated to be 23%.
In addition to atorvastatin, which is the most appropriate medication to prescribe?
A. Amlodipine
B. Bendroflumethiazide C. Doxazosin
D. Losartan potassium E. Ramipril

A

A. Amlodipine

183
Q

The association between low birth weight babies and maternal smoking during pregnancy was studied by obtaining smoking histories from women at the time of their first prenatal visit, then assessing birth weight at delivery and analysing according to the smoking histories.
Which is the best description of this type of study?
A. Case–control
B. Case series
C. Clinical trial
D. Cross-sectional
E. Prospective cohort

A

E. Prospective cohort

184
Q

A 60 year old woman is brought in after found drowsy and confused by her daughter. She has been unwell for the past 2–3 days with diarrhoea and vomiting. She has a history of bipolar disorder. Her regular medication includes lithium, risperidone and co-codamol.
Her temperature is 37.3°C, pulse rate 94 bpm, BP 122/70 mmHg, respiratory rate 14 breaths per minute and oxygen saturation 99% breathing high-flow oxygen. Her GCS score is 12/15. She has coarse tremor in her arms and jerking movements of her legs.
Which is the most likely diagnosis?
A. Hypernatraemia
B. Lithium toxicity
C. Neuroleptic malignant syndrome
D. Opioid toxicity
E. Subdural haematoma

A

B. Lithium toxicity

185
Q

A 42 year old woman has a three month history of weight loss, insomnia, and palpitations. She has a fine resting tremor and bilateral proptosis.
Which is the underlying pathological mechanism?
A. Antibody directed against the thyroid stimulating hormone receptor
B. Antibody directed against thyroid peroxidase
C. Autonomous activity of thyroid follicular cells
D. Destruction of thyroid cells by lymphocytes
E. Excess production of thyroid stimulating hormone

A

A. Antibody directed against the thyroid stimulating hormone receptor

186
Q

A 53 year old woman has 6 months of worsening tiredness.
She has jaundice, xanthelasma and 7 cm non-tender hepatomegaly.
Investigations: INR 1.2 (1.0)
ALT 60 IU/L (10–50)
ALP 302 IU/L (25–115)
Bilirubin 50 μmol/L (<17)
Antinuclear antibodies 1:40 (negative at 1:20) Antimitochondrial antibodies 1:320 (negative at 1:20)
Ultrasound scan of abdomen hepatosplenomegaly, no biliary dilatation Which is the most appropriate treatment?
A. Azathioprine
B. Lamivudine
C. Prednisolone
D. Thiamine
E. Ursodeoxycholic acid

A

E. Ursodeoxycholic acid

187
Q

A 91 year old woman is admitted from a nursing home with a 3 day history of vomiting and diarrhoea.
Her pulse rate is 110 bpm. Her BP is 116/66 mmHg lying in bed, with a postural BP drop of 30 mmHg when sitting.
Investigations:
Sodium Potassium Bicarbonate Urea Creatinine
130 mmol/L 4.0 mmol/L
20 mmol/L 25.6 mmol/L
(135–146) (3.5–5.3)
(22–29) (2.5–7.8)
177 μmol/L
Which is the most appropriate initial intravenous fluid?
(60–120)
A. 0.9% sodium chloride
B. 1.4% sodium bicarbonate
C. 1.8% sodium chloride
D. 4% glucose, 0.18% sodium chloride
E. 5% glucose

A

A. 0.9% sodium chloride

188
Q

A 30 year old man has three months of intermittent, but worsening headaches. He is not taking any regular medications aside from paracetamol and ibuprofen as required. His BP is 220/130 mmHg.
Investigations:
Sodium 144 mmol/L (135–146)Potassium 3.0 mmol/L (3.5–5.3) Urea 7.0 mmol/L (2.5–7.8)
Creatinine 92 μmol/L (60–120)
Which is the most likely underlying diagnosis?
A. Addison’s disease
B. Chronic kidney disease
C. Cushing’s disease

D. Phaeochromocytoma
E. Primary aldosteronism (Conn’s syndrome)

A

E. Primary aldosteronism (Conn’s syndrome)

189
Q

A 24 year old man has acute shortness of breath and lightheadedness. He was admitted 24 hours earlier following a car accident. He had multiple pelvic fractures and compound fractures of both tibia. These required surgical fixation. He is on intravenous morphine via a patient-controlled analgesia device, prophylactic low molecular weight heparin, intravenous flucloxacillin and intravenous 0.9% saline at 120 mL/hour.He is confused and disorientated. His temperature is 36.4°C, pulse rate 100 bpm, BP 110/60 mmHg, respiratory rate 30 breaths per minute and oxygen saturation 85% breathing 4 L/min oxygen via nasal prongs. His chest is clear.
Which is the most likely diagnosis?
A. Cardiac tamponade
B. Fat embolism syndrome
C. Opiate toxicity
D. Pulmonary embolus
E. Subdural haematoma

A

B. Fat embolism syndrome

190
Q

A 46 year old man has a cardiac arrest in the Emergency Department after an episode of chest pain. He remains in ventricular fibrillation after three DC shocks, and he is treated with a bolus of intravenous adrenaline/epinephrine.
Which other drug treatment should be administered at the same time?
A. Alteplase
B. Amiodarone hydrochloride
C. Atropine sulfate
D. Lidocaine
E. Magnesium sulfate

A

B. Amiodarone hydrochloride

191
Q

An 18 year old man has 3 weeks of malaise, fever and headaches and 1 week of a sore throat.
He has large tonsils with exudate, a petechial rash on the palate, and axillary and inguinal lymphadenopathy. His temperature is 37.6°C, pulse rate 84 bpm, BP 120/82 mmHg and respiratory rate 12 breaths per minute.
Investigations: White cell count Lymphocytes
ALT
AST
Alkaline phosphatase Bilirubin
11.2 × 109/L (3.8–10.0) 5.5 × 109/L (1.1–3.3)
72 IU/L 45 IU/L
91 IU/L 16 μmol/L
(10–50) (10–40)
(25–115) (<17)
Which is the most appropriate investigation to confirm the diagnosis?
A. Anti-streptolysin O titre
B. Blood cultures
C. Epstein–Barr virus serology
D. Hepatitis A serology
E. Throat swab

A

C. Epstein–Barr virus serology

192
Q

A 74 year old man is brought to the Emergency Department by his son after falling down the stairs. He has no pain. He has a history of atrial fibrillation and takes apixaban.
He has significant bruising to the left side of his face and left arm. His pulse rate is 80 bpm, irregular, BP 150/95 mmHg and oxygen saturation 96% breathing air. His GCS score is 14.

Which is the most appropriate next step in management?
A. Cervical spine immobilisation
B. Chest X-ray
C. CT of head
D. Intravenous prothrombin complex E. Intravenous vitamin K

A

A. Cervical spine immobilisation

193
Q

A 17 year old boy has repeated episodes characterised by a funny ‘racing’ sensation in his abdomen, followed by loss of awareness. His girlfriend describes that he has a vacant stare and waves his left arm around in a writhing manner during these attacks.
Which is the most likely site of origin of these episodes?
A. Cerebellum
B. Right frontal lobe
C. Right occipital lobe
D. Right parietal lobe
E. Right temporal lobe

A

E. Right temporal lobe

194
Q

A 45 year old woman is feeling unwell with a severe sore throat. She has schizophrenia and started taking clozapine 4 months ago.
Her temperature is 39.0°C. She has an ulcerated throat with tonsillar exudate.
Blood cultures have been sent.
What additional investigation is most likely to establish the underlying cause?
A. Clozapine concentration B. Creatine kinase
C. Full blood count
D. Lumbar puncture
E. Throat swab

A

C. Full blood count

195
Q

A 34 year old man has three days of fever, sweats and productive cough, bringing up brown sputum.
His temperature is 38.8°C. he has bronchial breathing over the right mid-zone. His chest X-ray is shown (see image).
<please insert picture MSCAA-050313-5841_figure_1.jpg. In your exported file, save and unpack the folder with images to be able to insert the image above this caption.>
Which is the most likely pathogen?
A. Haemophilus influenzae
B. Legionella pneumophila
C. Mycoplasma sp.
D. Staphylococcus aureus
E. Streptococcus pneumoniae

A

E. Streptococcus pneumoniae

196
Q

A 38 year old man has a 3 month history of a cough with bloodstained sputum. A sputum analysis is positive for Mycobacterium tuberculosis. He is treated with quadruple antituberculous therapy. Two weeks later he reports that he is passing orange urine.
Which is the medication most likely to be responsible for his orange urine?
A. Ethambutol
B. Isoniazid
C. Pyrazinamide
D. Pyridoxine
E. Rifampicin

A

E. Rifampicin

197
Q

A 46 year old woman attends the Emergency Department with fever, headache and confusion, which have developed over several hours. She finds it impossible to lift her head from the pillow and resists your attempts to feel her neck.
Her temperature is 38.1°C, pulse rate 105 bpm and BP 110/60
mmHg. Her GCS score is 14. A CT scan of her head is normal. A lumbar puncture is performed.
Which are the most likely observations in the cerebrospinal fluid?
A. High pressure, normal protein, excess red cells
B. High pressure, raised protein, excess neutrophils
C. Normal pressure, normal protein, excess lymphocytes
D. Normal pressure, raised protein, excess neutrophils
E. Normal pressure, normal protein, no cells

A

B. High pressure, raised protein, excess neutrophils

198
Q

A 51 year old woman presents to her GP with 6 months of dry mouth and gritty eyes. She has had vitiligo for the last 10 years.
Both parotid glands are mildly enlarged. Which is the most likely diagnosis?
A. Graves’ disease
B. Pleomorphic adenoma
C. Sarcoidosis
D. Sialolithiasis (salivary gland calculi)
E. Sjögren’s syndrome

A

E. Sjögren’s syndrome

199
Q

A 62 year old man has one year of intermittent heartburn and difficulty in swallowing.
An endoscopic biopsy of the oesophagus 5 cm above the anatomical gastro- oesophageal junction is reported as showing ‘columnar epithelium containing goblet cells and Paneth cells’.
Which is the most appropriate pathological description of the features noted?
A. Hyperplasia
B. Hypertrophy
C. Intestinal metaplasia
D. Intraepithelial neoplasia
E. Squamous metaplasia

A

C. Intestinal metaplasia

200
Q

A 64 year old man is admitted to the Emergency Department with acute dyspnoea. His exercise tolerance has been limited by breathlessness for several years.
Investigations:
Arterial blood gas breathing air:
pH
PCO2 Bicarbonate Base excess
7.20
8.1 kPa
23 mmol/L –4 mmol/L
(7.35–7.45)PO2 (4.6–6.4)
(22–30) (–2 to +2)
8 kPa
(11–15)
Which is the best interpretation of the arterial blood result?
A. Acute type 1 respiratory failure
B. Acute type 2 respiratory failure
C. Chronic type 1 respiratory failure
D. Chronic type 2 respiratory failure
E. Metabolic acidosis

A

B. Acute type 2 respiratory failure

201
Q

A73yearoldmancollapsesonthesurgicalward24hoursafterhavinga sigmoid volvulus reduced by sigmoidoscopy.
He has no pulse and an ECG shows asystole. Chest compressions and ventilation are started. The cardiac arrest team are with the patient.
Which is the most appropriate next step in management?
A. Cardiac defibrillation
B. Intravenous 0.9% sodium chloride
C. Intravenous adrenaline/epinephrine
D. Intravenous atropine sulfate
E. Transcutaneous pacing

A

C. Intravenous adrenaline/epinephrine

202
Q

A78yearoldwomanhaspaininbothshoulders,hipsandthighs.Sheisvery stiff on waking in the morning and takes 2–3 hours to loosen up. She finds getting dressed difficult.
Investigations:
Erythrocyte sedimentation rate 67 mm/hr (<20) CRP 87 mg/L (<5)
What is the most appropriate initial treatment?
A. Co-codamol B. Leflunomide C. Methotrexate D. Naproxen
E. Prednisolone

A

E. Prednisolone

203
Q

A36yearoldmanisrescuedfromahousefire.
He is alert and talking but has a dull headache. His pulse rate is 98 bpm, BP 139/86 mmHg, respiratory rate 22 breaths per minute and
oxygen saturation 100% breathing 15 L/min oxygen via a non-rebreather mask.
Which is the most appropriate parameter to measure?
A. Bicarbonate
B. Carboxyhaemoglobin
C. Haemoglobin
D. Lactate
E. Methaemoglobin

A

B. Carboxyhaemoglobin

204
Q

A60yearoldmanhas2daysofaswollen,painfulrightleg.Hehasahistory of hypertension and takes ramipril. He is otherwise well.
He has a swollen right leg. The remainder of the examination is normal.
Investigations: Haemoglobin White cell count Platelets Creatinine Calcium
ALT ALP APTT PT
140 g/L 8.0 × 109/L
340 × 109/L 94 μmol/L
2.5 mmol/L 30 IU/L
99 IU/L
30 seconds 12 seconds
(130–175) (3.8–10.0) (150–400)
(60–120) (2.2–2.6) (10–50)
(25–115) (22–41)
(10–12)
Urinalysis: normal
Chest X-ray: normal
Venous duplex ultrasound scan: thrombus in superficial femoral vein
Which is the most appropriate additional investigation?
A. CT of abdomen and pelvis
B. Serum carcinoembryonic antigen C. Serum prostate specific antigen D. Serum protein electrophoresis
E. Ultrasonography of abdomen

A

A. CT of abdomen and pelvis

205
Q

A65yearoldwomanhadamechanicalaorticvalvereplacementand coronary revascularisation 3 days ago. She is being treated with dalteparin sodium. She is also taking aspirin long term.
Which is the most appropriate long-term patient management?
A. Apixaban
B. Clopidogrel
C. Continue dalteparin sodium
D. Rivaroxaban
E. Warfarin sodium

A

E. Warfarin sodium

206
Q

Theassociationbetweenmaternalsmokingduringpregnancyandlow birthweight can be studied by obtaining smoking histories from pregnant women at the time of first prenatal visit, then assessing birthweight at delivery and analysing the data according to the smoking histories.
Which is the best description of this type of study?
A. Case–control
B. Cross-sectional
C. Prospective cohort
D. Randomised clinical trial
E. Retrospective cohort

A

C. Prospective cohort

207
Q

An18yearoldwomanhashadachronicskinconditionforseveralyears.She has noticed that she gets well-defined areas of scale formation at the sites of minor skin injuries, such as scratches or insect bites, typically when the injuries are healing. The scaling can persist for several weeks or months.
What is the most likely underlying skin condition?
A. Acne vulgaris
B. Eczema
C. Psoriasis
D. Seborrhoeic dermatitis E. Vitiligo

A

C. Psoriasis

208
Q

A64yearoldmanhasvomitingandseverechestpainaftereatingalarge meal.
His temperature is 37.6°C, pulse rate 130 bpm, BP 95/50 mmHg and respiratory rate 30 breaths per minute. There is palpable subcutaneous emphysema on the left side of his neck.
Which is the most likely diagnosis?
A. Diaphragmatic rupture
B. Mallory–Weiss tear
C. Necrotising fasciitis
D. Oesophageal rupture
E. Spontaneous pneumohaemothorax

A

D. Oesophageal rupture

209
Q

A54yearoldwomanhaspolyuriaandthefeelingthatsheneedstodrink continuously. She drinks at least 1 litre of water before bedtime and gets up three to four times during the night to pass urine. She has another glass of water each time that she gets up.
Investigations: Sodium Potassium
Urea
Creatinine Calcium
Fasting glucose Serum osmolality Urinary osmolality
140 mmol/L 4.1 mmol/L 4.5 mmol/L
86 μmol/L 2.56 mmol/L 4.8 mmol/L
(135–146) (3.5–5.3) (2.5–7.8) (60–120) (2.2–2.6)
(3.0–6.0) 295 mOsmol/kg (285–295)
86 mOsmol/kg (100–1000)
After 8 hours of a water deprivation test, the serum osmolality is 308 mOsmol/kg and the urinary osmolality is 152 mOsmol/kg.
Following the administration of desmopressin, the serum osmolality is 286 mOsmol/kg and the urinary osmolality is 660 mOsmol/kg.
Which is the most appropriate next investigation?
A. CT scan of thorax, abdomen and pelvis
B. MR scan of pituitary
C. Oral glucose tolerance test
D. Technetium-99 Sestamibi parathyroid scan
E. Supervised fluid restriction and daily weights

A

B. MR scan of pituitary

210
Q

A 30 year old woman has irregular periods, decreased libido and galactorrhoea.
Visual field examination is normal.
Investigations:
Prolactin: 5000 mU/L (100–500)
Pregnancy test: negative
MR scan of pituitary shows a 4-mm mass in the sella turcica. Which is the most appropriate management?
A. Cabergoline
B. Dexamethasone
C. Octreotide
D. Radiotherapy
E. Transsphenoidal surgery

A

A. Cabergoline

211
Q

A 23 year old man is admitted to the acute surgical ward with appendicitis and is prepared for theatre. Although he has not eaten for 24 hours, he has been vomiting on and off all day.
Which airway device provides protection for the lungs from regurgitated stomach contents?
A. Guedel (oral) airway
B. i-gel ® (supraglottic) airway C. Laryngeal mask airway
D. Nasopharyngeal airway
E. Tracheal tube

A

E. Tracheal tube

212
Q

A 32 year old man is referred to a gastroenterology clinic with hepatomegaly. He has a history of type 2 diabetes that is diet controlled. He is taking no regular medication. He is a non-smoker and drinks approximately 16 units of alcohol per week. He is married with no children.
Cardiovascular and respiratory examinations are normal. His abdomen is soft, with a 3 cm palpable liver edge. His BMI is 23 kg/m2(18–25).
Investigations:
Albumin ALT ALP Bilirubin Ferritin CRP
38 g/L
90 IU/L
112 IU/L
15 μmol/L (<17)
710 μg/L (12–200) 6 mg/L (<5)
(35–50) (10–50) (25–115)
Which is the most appropriate next investigation?
A. Hepatitis C serology B. Liver biopsy
C. Reticulocyte count D. Serum γGT
E. Transferrin saturation

A

E. Transferrin saturation

213
Q

A 30 year old woman has severe headache 24 hours after a spinal anaesthetic.
Her temperature is 37.1°C, pulse rate 90 bpm and BP 120/80 mmHg. Which is the most likely diagnosis?
A. Low pressure headache
B. Meningitis
C. Migraine
D. Subarachnoid haemorrhage
E. Subdural haemorrhage

A

A. Low pressure headache

214
Q

A 27 year old man is brought to the Emergency Department with left-sided chest pain of sudden onset that is worse on taking a deep breath.
His temperature is 36.8°C, pulse rate 126 bpm, BP 108/60 mmHg, respiratory rate 28 breaths per minute and oxygen saturation 94% breathing air.
Investigations:
ECG: sinus tachycardia
Which is the most appropriate next investigation?
A. Chest X-ray
B. CT pulmonary angiography
C. D dimers
D. Echocardiography
E. Ventilation/perfusion isotope lung scan

A

A. Chest X-ray

215
Q

A healthy 23 year old man is scheduled to undergo an elective arthroscopy of his knee. He is to have a general anaesthetic for the operation and asks the pre-operative assessment nurse how long he needs to fast beforehand.
Which are the most appropriate fasting times for clear liquids and solids respectively?
A. 2 h for clear liquids, 2 h for solids B. 2 h for clear liquids, 6 h for solids C. 2 h for clear liquids, 12 h for solids D. 6 h for both clear liquids and solids E. 6 h for clear liquids, 12 h for solids

A

B. 2 h for clear liquids, 6 h for solids

216
Q

A 28 year old woman presents to her GP with a neck lump that she noticed incidentally when rubbing her neck.
There is a smooth, non-tender 1.5 cm mobile lump within the thyroid gland.
Investigations:
TSH 2.3 mU/L (0.3–4.2) Free T4 17 pmol/L (9–25)
Which is the most appropriate next investigation?
A. CT of neck
B. No further investigations
C. Thyroid antibodies
D. Thyroid scintigraphy
E. Ultrasonography of neck

A

E. Ultrasonography of neck

217
Q

A 78 year old man has type 2 diabetes. His clinician does not invite him to join an internet-based self-monitoring programme because she considers him to be too old to engage with it effectively.
What is the most appropriate description of the clinician’s approach?
A. Bias
B. Discrimination C. Inequity
D. Prejudice
E. Stereotyping

A

B. Discrimination

218
Q

A 47 year old man with hypertension attends for annual review. He takes ramipril (10 mg once daily).
His BP is 138/78 mmHg. Investigations:
Sodium Potassium Urea Creatinine
139 mmol/L (135–146) 6.2 mmol/L (3.5–5.3) 5.0 mmol/L (2.5–7.8)
90 μmol/L (60–120)
Which is the most appropriate immediate action?
A. Add indapamide
B. Advise low potassium diet
C. Change ramipril to amlodipine D. Reduce dose of ramipril
E. Repeat urea and electrolytes

A

E. Repeat urea and electrolytes

219
Q

A 33 year old woman has 4 months of joint pain and stiffness, predominantly affecting her feet. This is worst in the morning and gradually improves through the day. She feels tired all the time but reports no other health problems.
Which investigation would confirm the most likely diagnosis?
A. Anti-cyclic citrillinated peptide antibody
B. Anti-double-stranded DNA antibodies
C. Antinuclear antibody
D. CRP
E. Rheumatoid factor

A

A. Anti-cyclic citrillinated peptide antibody

220
Q

A 27 year old man has severe central chest pain. He admits to using cocaine shortly before the onset of the chest pain, but says that he had used it on only two previous occasions.
He is distressed and sweating. His pulse rate is 115 bpm and BP 118/68 mmHg. An ECG shows sinus tachycardia with ST elevation in the lateral leads, and several ventricular ectopics.
Which is the mechanism by which cocaine has caused this acute episode?
A. Blockade of myocyte repolarisation
B. Coronary artery spasm
C. Enhanced platelet aggregation
D. Increased systemic vascular resistance
E. Rupture of pre-existing arterial plaques

A

B. Coronary artery spasm

221
Q

An 18 year old man, who was born in the UK, develops drowsiness and confusion 2 days after returning from visiting his grandparents in Malawi. Over the past week he has had recurrent episodes of high fever.
Investigations: Haemoglobin White cell count Platelets
92 g/L
3.2 × 109/L
(130–175) (3.8–10.0) (150–400)
Blood film
Which is the most likely causative organism?
184 × 109/L parasites visible
A. Plasmodium falciparum B. Plasmodium malariae C. Plasmodium vivax
D. Trypanosoma brucei
E. Trypanosoma cruzi

A

A. Plasmodium falciparum

222
Q

A 31 year old man visits his GP with a painless lump in his scrotum.
There is a well-defined, non-tender spherical 1 cm mass on the right side of
the scrotum. It is superior to the testis and transilluminates. Which is the most likely diagnosis?
A. Abscess
B. Epididymal cyst
C. Hydrocoele
D. Inguinal hernia
E. Testicular tumour

A

B. Epididymal cyst

223
Q

A 68 year old man has 3 days of worsening vomiting and abdominal pain. He has not passed any stool for 3 days. He has a history of a right hemicolectomy for Dukes’ A (T1, N0) bowel cancer 6 months ago.
He is dehydrated and his abdomen is distended. Which is the most likely diagnosis?
A. Adhesional small bowel obstruction
B. Anastomotic leak
C. Cholecystitis
D. Pancreatitis
E. Tumour recurrence

A

A. Adhesional small bowel obstruction

224
Q

A 78 year old patient is admitted with chronic oropharyngeal dysphagia. He has left ventricular systolic dysfunction from ischaemic heart disease. He is breathless on exertion, particularly when climbing stairs. He is being prepared for a percutaneous endoscopic gastrostomy feeding tube. The passage of a nasogastric tube has been unsuccessful, and he is having nil by mouth.
He weighs 70 kg. His pulse rate is 72 bpm and BP 132/80 mmHg.
Which is the correct volume of maintenance fluids (in mL) to prescribe for the next 24 hours?
A. 1750 B. 2450 C. 2800 D. 3000 E. 3250

A

A. 1750

225
Q

A 75 year old woman is reviewed 4 days after a fractured neck of femur repair. She has been agitated and upset, particularly at night. She has punched nurses and keeps trying to leave the ward. She has seen strange men in black capes entering the ward and believes that they are controlling the hospital. When she was seen in the memory clinic 6 months ago, she was found to have mild cognitive impairment.
What is the most appropriate treatment?
A. Chlordiazepoxide hydrochloride B. Chlorpromazine hydrochloride C. Donepezil hydrochloride
D. Haloperidol
E. Memantine hydrochloride

A

D. Haloperidol

226
Q

A 72 year old woman has 6 months of increasing pain at the base of her right thumb. She is having difficulty opening jars and sewing. She is otherwise well. No other joints are painful. She is taking regular analgesia.
The first carpometacarpal joint is swollen and tender, with reduced opposition of the thumb.
Which is the most likely diagnosis?
A. De Quervain’s tenosynovitis
B. Gout
C. Osteoarthritis
D. Rheumatoid arthritis
E. Septic arthritis

A

C. Osteoarthritis

227
Q

A 62 year old man has acute breathlessness with a weak cough, following a recent viral upper respiratory infection. Over the past 4 months, he has
had double vision, limb weakness and slurred speech when tired.
His respiratory rate is 18 breaths per minute and oxygen saturation 96% breathing air. He is sweating and using his accessory muscles of inspiration.
Which is the most appropriate test to monitor his respiratory function?
A. Arterial blood gas
B. FEV 1
C. FVC
D. Peak expiratory flow rate
E. RatioofFEV1toFVCA 62 year old man has acute breathlessness with a weak cough, following a recent viral upper respiratory infection. Over the past 4 months, he has
had double vision, limb weakness and slurred speech when tired.
His respiratory rate is 18 breaths per minute and oxygen saturation 96% breathing air. He is sweating and using his accessory muscles of inspiration.
Which is the most appropriate test to monitor his respiratory function?
A. Arterial blood gas
B. FEV 1
C. FVC
D. Peak expiratory flow rate
E. RatioofFEV1toFVC

A

C. FVC

228
Q

A 40 year old man has 4 days of left flank pain associated with fever, nausea and vomiting.
His temperature is 39.6°C, pulse rate 118 bpm and BP 90/40 mmHg. Imaging shows an obstructing proximal left ureteric stone with severe hydronephrosis.
He is treated with intravenous antibiotics and intravenous fluids. Which is the most appropriate next step in management?
A. Lithotripsy
B. Nephrostomy
C. Retrograde pyelography
D. Ureteric stent
E. Urethral catheter

A

B. Nephrostomy

229
Q

A 19 year old man has 2 days of right-sided chest pain and breathlessness on exertion. He smokes cannabis and takes cocaine.
His temperature is 36.4°C, pulse rate 108 bpm, BP 112/80 mmHg, respiratory rate 24 breaths per minute and oxygen saturation 94% breathing air. His trachea is central. He has reduced breath sounds at the right apex.
What is the most likely diagnosis?
A. Acute coronary syndrome B. Coronary artery spasm C. Pneumonia
D. Pneumothorax
E. Pulmonary embolism

A

D. Pneumothorax

230
Q

An 80 year old woman is admitted to the Emergency Department after being found collapsed at home. She has central chest pain.
Her pulse rate is 30 bpm, BP 70/40 mmHg and respiratory rate 26 breaths per minute. Her 12-lead ECG shows sinus bradycardia with no evidence of myocardial ischaemia.
Which is the most appropriate initial treatment?
A. Adrenaline/epinephrine
B. Atropine sulfate
C. Dobutamine
D. Normal saline
E. Permanent cardiac pacemaker

A

B. Atropine sulfate

231
Q

A 62 year old man develops acute pain, redness, swelling and warmth of his right first metatarsophalangeal joint. He has a history of gout and hypertension. His medications are allopurinol, amlodipine and ramipril.
His eGFR is >60 mL/min/1.73 m2 (>60).
Which is the most appropriate next step in his management?
A. Change allopurinol to febuxostat
B. Start naproxen
C. Stop allopurinol
D. Stop amlodipine
E. Stop ramipril

A

B. Start naproxen

232
Q

A previously healthy 10 year old boy has deafness of new onset. He has a history of a recent respiratory tract infection.
Tuning fork tests show:
a) when the tuning fork is placed in the middle of his forehead, he hears the tone loudest in his right ear.
b) when the tuning fork is held in front of his right external auditory meatus it is quieter than when it is placed on the bone behind the same ear.
c) when the tuning fork is held in front of the left external auditory meatus the sound is louder than when it is placed on the bone behind the same ear.
Which ear(s) is/are affected and which type of hearing loss is this?
A. Bilateral mixed deafness
B. Left conductive deafness
C. Left sensorineural deafness D. Right conductive deafness
E. Right sensorineural deafness

A

D. Right conductive deafness

233
Q

A 55 year old man has 2 days of painful red swelling of his left lower leg. He has a history of type 2 diabetes mellitus and takes metformin.
His temperature is 37.6°C. He has a tender erythematous area extending from the ankle to the proximal calf.
What is the most likely causative organism?
A. Bacteroides species
B. Proteus mirabilis
C. Pseudomonas aeruginosa D. Staphylococcus epidermidis E. Streptococcus pyogenes

A

E. Streptococcus pyogenes

234
Q

A 55 year old woman has a tender, erythematous, swollen hard cord in the long saphenous vein distribution in her calf. She has a longstanding history of bilateral varicose veins.
An ultrasound scan shows superficial thrombophlebitis without deep vein thrombosis.
Which is the most appropriate treatment?
A. Dipyridamole
B. Flucloxacillin
C. Naproxen
D. Paracetamol
E. Rivaroxaban

A

C. Naproxen

235
Q

A 64 year old woman is due to undergo an arthroscopy of her knee. She has type 2 diabetes and takes metformin (500 mg twice daily) and gliclazide (80 mg each morning).
Her glycated haemoglobin is 54 mmol/mol (20–42). She is scheduled first on the morning day case list and is asked to fast from midnight the previous night.
Which is the most appropriate plan for managing her diabetic medication?
A. Continue both drugs and start a variable-rate insulin infusion
B. No change to usual medication
C. Omit both drugs and start a variable-rate insulin infusion
D. Omit gliclazide and continue metformin
E. Omit metformin and continue gliclazide

A

D. Omit gliclazide and continue metformin

236
Q

A 67 year old man has difficulty walking. He states that he has to raise his left leg higher in the air than normal to avoid scraping his toes on the ground when he walks.
When he raises the left foot from the floor, the ankle assumes a plantar-flexed position with the toes directed towards the floor.
Which nerve is most likely to be affected?
A. Common peroneal
B. Medial plantar
C. Saphenous
D. Superficial peroneal
E. Tibial

A

A. Common peroneal

237
Q

A 35 year old woman has 6 months of cyclical pain in both breasts. She has recently noticed a lump in the right breast. There is diffuse nodularity of the axillary tails of both breasts with a discrete 20 mm mass in the upper outer quadrant of the right breast. Fine needle aspiration is performed and 5 mL of brown fluid is removed, with disappearance of the mass. Cytology shows cellular debris with no malignant cells.
What is the most likely diagnosis?
A. Breast abscess
B. Breast carcinoma C. Fat necrosis
D. Fibroadenoma
E. Fibrocystic disease

A

E. Fibrocystic disease

238
Q

A 70 year old man has a brief episode of twitching that starts in his left hand and spreads up the arm over 2 minutes, then stops. His arm feels weak for an hour afterwards. He had an ischaemic stroke affecting his left side 6 months ago with good functional recovery. He has a history of type 2 diabetes mellitus and is taking clopidogrel, metformin, ramipril and simvastatin. He is anxious about a further stroke.
There is no weakness on neurological examination. Which is the most likely diagnosis?
A. Functional episode
B. Hypoglycaemia
C. Migraine
D. Partial seizure
E. Right hemisphere transient ischaemic attack

A

D. Partial seizure

239
Q

A 63 year old woman has 4 months of abdominal bloating, fatigue and nausea.
She is found to have ovarian cancer. Staging CT is performed to look for lymphatic spread and metastatic disease.
To what regional lymph nodes is her tumour most likely to spread initially?
A. Deep inguinal nodes B. External iliac nodes C. Internal iliac nodes D. Para-aortic nodes
E. Superficial inguinal nodes

A

D. Para-aortic nodes

240
Q

A 52 year old woman has had three episodes of severe epigastric pain associated with vomiting over the past 3 months. The episodes occurred following eating and lasted for about 1 hour. She has type 2 diabetes mellitus and takes metformin.
Abdominal examination is normal. Her BMI is 35 kg/m2 (18–25).
Investigations:
ALT 15 IU/L ALP 71 IU/L Bilirubin 9 μmol/L
(10–50) (25–115)
(<17)
Ultrasound scan of abdomen: single 2-cm gallstone in gallbladder, common bile duct normal, evidence of fatty liver.
Which is the most appropriate management?
A. Endoscopic retrograde cholangiopancreatography B. Laparoscopic cholecystectomy
C. MR cholangiopancreatography
D. Open cholecystectomy
E. Ursodeoxycholic acid

A

B. Laparoscopic cholecystectomy

241
Q

A 56 year old woman develops vertigo, nausea, vomiting and intense occipital headache of sudden onset. She is unable to walk without falling. She has a history of hypertension treated with ramipril.
Her temperature is 37.4°C, pulse rate 94 bpm, BP 146/92 mmHg, respiratory rate 12 breaths per minute and oxygen saturation 96% breathing air. She has multidirectional nystagmus and some clumsiness of her right arm.
Which is the most likely diagnosis?
A. Acute labyrinthitis
B. Benign paroxysmal positional vertigo
C. Cerebellar stroke
D. Ménière’s disease
E. Multiple sclerosis

A

C. Cerebellar stroke

242
Q

A 79 year old woman has 3 months of a left leg venous ulcer that is slowly healing.
Ankle–brachial pressure Indices are 0.9 on the left side and 1.1 on the right side (0.8–1.2).
Which is the most appropriate management?
A. Compression stockings
B. Diagnostic biopsy
C. Full-length graduated compression bandaging
D. No further management required
E. Repeat ankle–brachial pressure indices in 3 months

A

A. Compression stockings

243
Q

A 26 year old woman sustains a head injury in a motorcycle accident. Her eyes are closed, but she opens them when asked to. She is confused about what happened and about where she is, but attempts to talk about it. She is repeatedly attempting to remove the cannula from her right wrist.
What is her GCS?
A. 3 B. 7 C. 9 D. 12 E. 14

A

D. 12

244
Q

A 65 year old man has sudden pain and redness in his right eye. He also has a headache and nausea.
Visual acuity is 6/60 in the right eye. The eye is congested, with a hazy cornea and mid-dilated pupil.
Which is the most likely diagnosis?
A. Acute glaucoma
B. Conjunctivitis
C. Corneal ulcer
D. Scleritis
E. Uveitis

A

A. Acute glaucoma

245
Q

A 45 year old woman attends her GP surgery with symptoms of vaginal soreness, itching and discharge. She has had recurrent episodes of vaginal candidiasis over the past 4 months.
The labia minora are red and swollen. A diagnosis of vaginal candidiasis is made.
What is the most appropriate investigation at this stage?
A. Glycated haemoglobin
B. HIV test
C. Sexually transmitted infection screen D. Test her partner for candidiasis
E. Vaginal pH testing

A

A. Glycated haemoglobin

246
Q

A 68 year old man collapses when rising from a chair and is seen in the emergency department 45 minutes later. He is conscious but has reduced power in his left arm and leg (3/5 and 4/5 respectively) and is slurring his speech. He has a past medical history of COPD and hypertension. He smokes 10 cigarettes per day.
He has bilateral scattered wheeze and carotid bruits on auscultation. Which is the most appropriate initial radiological investigation?
A. Carotid ultrasonography
B. Cerebral angiography
C. CT cerebral venography
D. CT of head
E. MR imaging of brain

A

D. CT of head

247
Q

A 25 year old man sustains 40% full-thickness burns in a house fire. Despite intensive treatment, he becomes breathless and hypotensive. He develops a petechial rash.
His temperature is 38°C, pulse rate 110 bpm, BP 80/50 mmHg and oxygen saturation 96% breathing 40% oxygen.
Investigations: Haemoglobin White cell count Platelets
APTT
PT
Fibrinogen
Fibrinogen degradation products 137 mg/mL
Which is the most likely diagnosis?
A. Anaphylactic reaction to antibiotics
B. Disseminated intravascular coagulation C. Fat embolism
D. Immune thrombocytopenic purpura
E. Pulmonary embolism

A

B. Disseminated intravascular coagulation

248
Q

A 65 year old woman with newly diagnosed advanced lung cancer has 1
day of breathlessness and 1 week of progressive headache. Her pulse rate is 88 bpm, respiratory rate 20 breaths per minute and oxygen saturation 95% breathing 4 L/min oxygen via nasal prongs. She has a swollen face and neck and distended veins on her chest. Her chest is clear. CT scan of chest shows mediastinal lymphadenopathy compressing the superior vena cava.
Which is the most appropriate initial treatment?
A. Intravenous alteplase
B. Intravenous dexamethasone
C. Intravenous heparin infusion
D. Intravenous mannitol
E. Insert endovenous stent

A

B. Intravenous dexamethasone

249
Q

An 80 year old man presents to the Emergency Department with dizziness and melaena of recent onset. He has a metallic mitral valve and is taking aspirin and warfarin sodium. His pulse rate is 80 bpm and BP 122/70 mmHg.
Investigations: Haemoglobin White cell count Platelets
105 g/L (130–175) 7.0 × 109/L (3.8–10.0) 676 × 109/L (150–400)
INR 9.6 (1.0)
He is treated with intravenous vitamin K and is blood cross-matched. What is the most appropriate additional treatment?
A. Fresh frozen plasma
B. Pantoprazole
C. Protamine sulfate
D. Prothrombin complex concentrate E. Tranexamic acid

A

D. Prothrombin complex concentrate

250
Q

A 65 year old man has 3 weeks of progressive ankle oedema. He is a lifelong heavy smoker and drinks 12 units of alcohol per week.
His BP is 125/85 mmHg and oxygen saturation 98% breathing air. He has marked bilateral pitting ankle oedema.
Investigations:
Creatinine
Urinary protein: creatinine ratio Fasting glucose
Total cholesterol
Albumin
85 μmol/L 400 mg/mmol 5.7 mmol/L
9 mmol/L
20 g/L
(60–120) (<30) (3.0–6.0) (<5.0) (35–50)
He is treated with furosemide.
Which investigation is most likely to be diagnostic?
A. Chest X-ray
B. Renal arteriography
C. Renal auto-antibody screen D. Renal biopsy
E. Serum protein electrophoresis

A

D. Renal biopsy

251
Q

A67yearoldwomansuddenlylosesthevisioninherlefteye.Sheisingood health with no history of eye disease and is not taking any medication.
Her right eye is normal. The left has vision reduced to hand movements only. The left pupil reacts sluggishly to light. Her fundal photograph is shown (see image).
Which is the cause of her sudden loss of vision?
A. Branch retinal artery occlusion
B. Branch retinal vein occlusion
C. Central retinal artery occlusion
D. Central retinal vein occlusion
E. Cilioretinal vein occlusion

A

B. Branch retinal vein occlusion

252
Q

A78yearoldmandevelopsincreasingconfusiononthewardatnight.Heis wandering around naked asking for his wife, shouting, and threatening staff and patients. He was admitted this morning with 1 week of productive cough and temperature. He has a history of idiopathic Parkinson’s disease.
His temperature is 37.6°C, pulse rate 100 bpm, BP 132/71 mmHg and oxygen saturation 95% breathing air. His capillary blood glucose is 5 mmol/L.
Attempts to calm him with nursing measures do not improve the situation, and he begins hitting staff.
Which is the most appropriate treatment?
A. Amitriptyline
B. Haloperidol
C. Immediate release carbidopa-levodopa
D. Lorazepam
E. Risperidone

A

D. Lorazepam

253
Q

A78yearoldwomanhasbackpainandbilaterallegweakness.Shehadsurgery for breast cancer 14 years ago. She had a minor fall at home a few days ago.
She has weakness of hip flexion bilaterally. Which is the most likely diagnosis?
A. Lumbar disc prolapse
B. Mechanical back pain
C. Osteomalacia
D. Osteoporotic vertebral collapse
E. Spinal cord compression

A

E. Spinal cord compression

254
Q

A28yearoldmanisinvestigatedforpolyuriaandpolydipsia.Hehasbipolar disorder for which he has taken lithium for 2 years.
Initial investigations:
Sodium Potassium
Serum osmolality Urinary osmolality Fasting glucose Serum lithium
145 mmol/L (135–146) 3.9 mmol/L (3.5–5.3) 296 mOsmol/kg (285–295)
356 mosmol/kg 5.8 mmol/L 0.75 mmol/L
(100–1000) (3.0–6.0) (0.5–1.2)
Which is the most useful diagnostic investigation?
A. 24-h urinary cortisol test
B. Glucose tolerance test
C. Serum corrected calcium
D. Short Synacthen test
E. Water deprivation test

A

C. Serum corrected calcium

255
Q

A65yearoldwomanhadastroke2weeksagocausingrightarmweaknessand dysphasia. CT scan of head showed a left parietal lobe infarct. Her medication since the stroke includes aspirin and simvastatin.
Her pulse rate is 82 bpm and irregular.
Investigations:
ECG: atrial fibrillation, rate 68 bpm.
Which is the most appropriate long-term plan for secondary stroke prevention?
A. Apixaban
B. Aspirin
C. Aspirin and dipyridamole
D. Clopidogrel
E. Ticagrelor

A

A. Apixaban

256
Q

A49yearoldmanhas3daysofcoughandfever.Heundergoeshaemodialysis three times per week.
His temperature is 38.6°C, pulse rate 90 bpm, BP 122/80 mmHg, respiratory rate 30 breaths per minute and oxygen saturation 95% breathing 15 L/min oxygen via a non-rebreather mask. His JVP is 4 cm above the sternal angle. He has inspiratory crackles in the right mid and lower zone and left upper zone. His blood capillary glucose is 12 mmol/L.
Investigations: Sodium Potassium Urea Creatinine
131 mmol/L (135–146) 5.7 mmol/L (3.5–5.3) 16.7 mmol/L (2.5–7.8)
327 μmol/L (60–120) Chext X-ray: see image
Which is the most appropriate initial treatment?
A. Haemodialysis
B. Intravenous 10 mL 10% calcium gluconate
C. Intravenous 1000 mL 0.9% sodium chloride over 1 h
D. Intravenous co-amoxiclav and clarithromycin
E. Intravenous furosemide

A

D. Intravenous co-amoxiclav and clarithromycin

257
Q

A67yearoldmanisduetohaveaCTscanofchest,abdomenandpelviswith intravenous contrast as assessment for possible lymphoma. He had a renal transplant 5 years ago. His medication includes prednisolone, tacrolimus and lisinopril.
His BP is 131/86 mmHg.
Investigations:
Urea 12.9 mmol/L Creatinine 165 μmol/L
eGFR 39 mL/min/1.73 m2
(2.5–7.8) (60–120)
(>60)
Which is the most appropriate treatment to give before the scan?
A. Intravenous 0.9% sodium chloride infusion B. Intravenous acetylcysteine
C. Intravenous furosemide infusion
D. Intravenous hydrocortisone
E. Intravenous mannitol

A

A. Intravenous 0.9% sodium chloride infusion

258
Q

A78yearoldwomanisadmittedafterbeenfoundcollapsedathome.Shehas been lying on the floor overnight. She has a history of hypertension and takes amlodipine.
Her temperature is 35.8°C, pulse rate 88 bpm and irregular, and BP 102/60 mmHg.
Investigations: Sodium Potassium Urea Creatinine
Creatine kinase
136 mmol/L 5.8 mmol/L 20.9 mmol/L
180 μmol/L 870 U/L
(135–146) (3.5–5.3) (2.5–7.8) (60–120)
(25–175)
Urinalysis: glucose negative, ketones negative, blood 2+, protein 1+, leucocytes positive (catheter sample).
She has passed 60 mL of urine over the past 2 hours. Which is the most likely cause for her acute kidney injury?
A. Glomerulonephritis
B. Hypovolaemia
C. Renal arterial emboli
D. Rhabdomyolysis
E. Ureteric obstruction

A

B. Hypovolaemia

259
Q

A76yearoldwomanhasbecomeincreasinglyconfusedoverthepast2weeks. She has a history of hypertension and cardiac failure. She is taking bendroflumethiazide, doxazosin, amlodipine, ramipril and atenolol.
Her BP is 108/71 mmHg. Investigations:
121 mmol/L 3.5 mmol/L 10.0 mmol/L
105 μmol/L
Which medication is most likely to be responsible for her presentation?
A. Amlodipine
B. Atenolol
C. Bendroflumethiazide
D. Doxazosin
E. Ramipril

A

C. Bendroflumethiazide

260
Q

A 92 year old woman has severe neck, chest and back pain and tingling in her left hand following a mechanical fall. She has bruising around her right eye.
Investigations:
CT scan of head: no intracranial injury or bleed, mild small vessel disease; right orbital fracture
Chest X-ray: lung fields clear; left sided 4th rib fracture
Full blood count and clotting screen are normal. Which is the most appropriate next investigation?
A. Cervical spine X-ray
B. CT angiography
C. CT scan of chest
D. CT scan of neck
E. MR scan of brain

A

D. CT scan of neck

261
Q

A 24 year old woman attends her GP with 2 months of intermittent palpitations. She describes the episodes as feeling as if the heart stops for a second followed by a pounding sensation. She reports episodes two to three times per week lasting 5–10 minutes, most commonly when she is going to sleep. She is otherwise well. She has been taking the combined oral contraceptive pill for 2 years.
Her pulse rate is 68 bpm and BP 108/71 mmHg. Her heart sounds are normal.
Investigations:
ECG: sinus rhythm, rate 70 bpm.
Which is the most likely diagnosis?
A. Paroxysmal atrial fibrillation
B. Sinus arrhythmia
C. Sinus tachycardia
D. Supraventricular premature beats
E. Supraventricular tachycardia

A

D. Supraventricular premature beats

262
Q

A 64 year old woman develops an acutely painful right knee 3 weeks after a prosthetic right knee replacement. Her temperature is 38.2°C and pulse rate 103 bpm. Her right knee is inflamed, with serous fluid discharging from the lower end of the wound.
Which is the most likely causative organism?
A. Corynebacterium species
B. Enterococcus species
C. Pseudomonas aeruginosa
D. Staphylococcus aureus
E. Streptococcus pyogenes

A

D. Staphylococcus aureus

263
Q

A 52 year old man has 3 months of fatigue. He has a history of ulcerative colitis and takes mesalazine. He drinks 20 units of alcohol per week.
His temperature is 36.8°C and pulse rate 80 bpm. He has 3 cm hepatomegaly.
Investigations: Albumin 36 g/L ALT 65 IU/L ALP 580 IU/L Bilirubin 18 μmol/L γGT 230 IU/L
(35–50) (10–50) (25–115)
(<17) (9–40)
Ultrasound scan of abdomen: bile duct wall thickening and dilatation Which is the most appropriate next diagnostic investigation?
A. CT scan of abdomen
B. Endoscopic retrograde cholangiopancreatography
C. Liver biopsy
D. MR cholangiopancreatography
E. Percutaneous transhepatic cholangiography

A

D. MR cholangiopancreatography

264
Q

A 60 year old woman has 6 weeks of a cough productive of blood streaked sputum.
Investigations:
CT scan of chest: mass in left lower lobe
Needle biopsy: nuclei that are enlarged, hyperchromatic and pleomorphic. Which is the most likely diagnosis?
A. Adenoma
B. Carcinoma
C. Hamartoma
D. Sarcoidosis
E. Tuberculosis

A

B. Carcinoma

265
Q

A 65 year old woman has an infective exacerbation of COPD.
Her temperature is 37.8°C, pulse rate 108 bpm, BP 100/75mmHg, respiratory rate 26 breaths per minute and oxygen saturation 88% breathing 15 L/minute oxygen via a non-rebreather mask. She is alert.
Investigations:
Arterial blood gas on 15 L/min oxygen
pH
PO2
PCO2 Bicarbonate Lactate
7.28 7.2 kPa
8.9 kPa 31.3 mmol/L
(7.35–7.45) (11–15)
(4.6–6.4) (22–30)
1.2 mmol/L
Which is the most appropriate next management option?
(1–2)
A. Continuous positive airway pressure
B. Invasive ventilation
C. Nasal high flow oxygen
D. Nasopharyngeal airway
E. Non-invasive ventilation

A

E. Non-invasive ventilation

266
Q

A 79 year old man attends the Emergency Department with 2 hours of chest pain and light-headedness.
His ECG is shown (see image).
Which is the most likely explanation for the ECG findings?
A. Aortic dissection
B. Hyperkalaemia
C. Myocardial infarction
D. Pericarditis
E. Pulmonary embolism

A

C. Myocardial infarction

267
Q

A 50 year old woman has acute onset of shortness of breath. She underwent a laparoscopic cholecystectomy 10 days ago.
Her temperature is 37.4°C, pulse rate 104 bpm, BP 122/80 mmHg, respiratory rate 24 breaths per minute and oxygen saturation 94% breathing air. Her chest is clear. She has minimal tenderness over the right hypochondrium.
Which is the most likely diagnosis?
A. Myocardial infarction
B. Pancreatitis
C. Pneumonia
D. Pulmonary embolus
E. Subphrenic abscess

A

D. Pulmonary embolus

268
Q

A 39 year old man has had fever, chills and generalised weakness for 1 month. He has a history of systolic heart murmur. He is an intravenous drug user.
Investigations:
White cell count 15 × 109/L (3.0–10.0) Erythrocyte sedimentation rate 55 mm/hr (<20)
Blood cultures are awaited.
Which further investigation will help to establish the diagnosis?
A. Chest X-ray
B. ECG
C. Nasal swabs
D. Transoesophageal echocardiogram
E. Urine dipstick analysis

A

D. Transoesophageal echocardiogram

269
Q

A 16 year old girl presents to the Emergency Department after an episode of loss of consciousness following a fall. She had consumed an excessive amount of alcohol at a party before the fall.
She has a dirty scalp wound and cannot remember recent events. Her pulse rate is 68 bpm, BP 110/80 mmHg and oxygen saturation 98% breathing air. She opens her eyes to command and is confused. Her capillary blood glucose is 6.0 mmol/L.
Her wound is cleaned and sutured.
Which is the most appropriate immediate management plan?
A. Admit and observe for 24 h
B. CT scan of head
C. Discharge with head injury instructions
D. Refer to neurosurgeon
E. X-ray of skull

A

B. CT scan of head

270
Q

A 47 year old woman has 1 year of increased urinary frequency, urgency and nocturia. She finds that she leaks urine if she is unable to get to the toilet promptly. She had three normal pregnancies with no complications.
The perineum appears normal and there is no uterine prolapse. Incontinence is not provoked by coughing or straining. Urinalysis is negative. A post-micturition ultrasound scan of bladder shows minimal residual urine volume.
Which is the most appropriate initial management plan?
A. Botulinum toxin type A injection into bladder wall
B. Oral duloxetine
C. Oral oxybutynin
D. Percutaneous sacral nerve stimulation
E. Topical oestrogen

A

C. Oral oxybutynin

271
Q

A 72 year old woman has 6 months of mild constipation.
Investigations: Haemoglobin Creatinine Calcium Phosphate
24 h urinary calcium Parathyroid hormone Vitamin D
A. Familial hypocalciuric hypercalcaemia B. Multiple myeloma
C. Primary hyperparathyroidism
D. Tertiary hyperparathyroidism
E. Vitamin D intoxication

A

C. Primary hyperparathyroidism

272
Q

A 52 year old man visits his GP for health screening. He is well. His pulse rate is 80 bpm and irregular, and BP 128/84 mmHg. Investigations:
Sodium Potassium Urea Creatinine
HbA1c
Thyroid function tests are normal. ECG shows atrial fibrillation, 76 bpm.
Which is the most appropriate treatment?
A. Apixaban
B. Aspirin
C. Diltiazem hydrochloride
D. No treatment
E. Warfarin sodium

A

D. No treatment

273
Q

An 80 year old man presents to the Emergency Department with 2 hours of epistaxis that has not stopped despite compression. He has a history of hypertension.
A bleeding site is visible in the anterior nasal cavity. His BP is 160/95 mmHg. Which is the most appropriate initial management option?
A. Anterior pack
B. Antihypertensive medication
C. Cautery
D. Cryotherapy
E. Ice pack

A

C. Cautery

274
Q

A 65 year old man has abdominal distension and vomiting 48 hours after a sigmoid colectomy with primary anastomosis for bowel cancer. He has not passed any flatus for 24 hours. He is taking regular paracetamol and as required intravenous morphine.
His abdomen is distended with tenderness over the wound but no rebound or guarding. There are no bowel sounds.
His temperature is 37.6°C, pulse rate 96 bpm and BP 122/85 mmHg. Which is the most appropriate initial management?
A. Give intravenous piperacillin with tazobactam
B. Give Microlax ® enema
C. Insert a nasogastric tube
D. Start regular intravenous morphine
E. Take to theatre for laparotomy

A

C. Insert a nasogastric tubeC. Insert a nasogastric tube

275
Q

A 23 year old man attends the sexual health clinic with a painful swelling in his groin and pain when opening his bowels. He had unprotected anal sex with a new male partner 4 weeks ago.
He has a perianal ulcer and tender inguinal lymphadenopathy. Which is the most likely diagnosis?
A. Genital herpes
B. Gonorrhoea
C. HIV seroconversion
D. Lymphogranuloma venereum
E. Secondary syphilis

A

D. Lymphogranuloma venereum

276
Q

A 63 year old man with COPD has had an increase in cough for 2 days. He now has pain of sudden onset in the right side of his chest radiating to the right shoulder and increasing breathlessness.
Chest X-ray: see image.
Which is the most likely cause of his acute deterioration?
A. Acute myocardial infarction
B. Diaphragmatic hernia
C. Lobar pneumonia
D. Pneumothorax
E. Pulmonary embolus

A

D. Pneumothorax

277
Q

A 25 year old man presents to the Emergency Department after vomiting a large quantity of blood.
At endoscopy, a deep ulcer is identified in the posterior wall at the junction of the first and second parts of the duodenum. There is a bleeding vessel in the base.
From which artery is the bleeding most likely to be arising?
A. Gastroduodenal artery B. Left gastric artery
C. Short gastric artery
D. Splenic artery
E. Superior mesenteric artery

A

A. Gastroduodenal artery

278
Q

A 27 year old woman has a long history of irregular menstruation. She does not take any medication. Her BMI is 29.4 kg/m2.
Investigations:
LH (pre-menopausal female)
FSH
Testosterone
Cortisol post 1 mg dexamethasone Prolactin
Which is the most likely diagnosis?
A. Androgen secreting tumour B. Cushing’s syndrome
C. Polycystic ovary syndrome D. Premature ovarian failure E. Prolactinoma

A

C. Polycystic ovary syndrome

279
Q

A 30 year old woman attends 3 months after a diagnosis of iron deficiency anaemia. She was advised to take oral iron supplements and has been taking these regularly. She has some looseness of her stools, and her periods are scanty on a combined oral contraceptive.
Investigations: Haemoglobin MCV
Ferritin
(115–165) (80–96)
A. Anti-gastric parietal cell
B. Antimitochondrial
C. Antinuclear
D. Anti-smooth muscle
E. Anti-tissue transglutaminase

A

E. Anti-tissue transglutaminase

280
Q

A 75 year old man is admitted with weakness in his legs. He has a squamous cell lung cancer treated by radiotherapy 18 months previously.
He is cachetic. He has 4 out of 5 power in hip flexion and knee flexion bilaterally. Sensation and reflexes are normal, and sphincter function is preserved. His BMI is 17 kg/m2. MR scan of spine shows destructive bony lesions of T12 and L2–L4 vertebral body.
Which treatment is most appropriate to preserve neurological function?
A. Chemotherapy
B. External beam radiotherapy
C. Intravenous bisphosphonates
D. Radiation brachytherapy
E. Surgical decompression of the spinal cord

A

B. External beam radiotherapy

281
Q

A 34 year old woman has headaches for 3 months. Her mother developed hypertension at the age of 38 years.
Her BP is 180/92 mmHg. Otherwise, examination, including fundoscopy, is normal.
Investigations:
Sodium Potassium Chloride Urea Creatinine
Plasma aldosterone: renin ratio Cortisol (9am)
Urinalysis is normal.
Which is the most likely diagnosis?
A. Essential hypertension
B. Glucocorticoid excess
C. Phaeochromocytoma
D. Primary hyperaldosteronism
E. Primary hypoadrenalism

A

D. Primary hyperaldosteronism

282
Q

A 20 year old man has 2 days of visible haematuria. Three days before the haematuria started, he developed tonsillitis and was treated with phenoxymethylpenicillin. He has no significant medical history.
His BP is 112/54 mmHg.
Investigations:
Urea 3.2 mmol/L (2.5–7.8) Creatinine 61 μmol/L (60–120)
Urinalysis: glucose negative, ketones negative, blood 3+, protein 3+, nitrites negative, leucocytes negative.
Which is the most likely cause of his haematuria?
A. Alport’s nephropathy
B. Drug reaction
C. Granulomatosis with polyangiitis D. IgA nephropathy
E. Postinfectious glomerulonephritis

A

D. IgA nephropathy

283
Q

A 38 year old man has 2 months of epigastric pain that radiates into his back. It is worse at night and sometimes wakes him up. It is better after eating. He has been very busy at work. He occasionally misses lunch, which worsens the pain. He has not lost weight. He smokes 10 cigarettes per day and drinks two bottles of wine per week.
Examination is normal. Which is the most likely diagnosis?
A. Cholecystitis
B. Chronic pancreatitis
C. Duodenal ulcer
D. Gastric ulcer
E. Gastric carcinoma

A

C. Duodenal ulcer

284
Q

A 26 year old woman has 12 months of back pain, located in the low lumbosacral region. She has intermittent bilateral thigh pain. The back pain disturbs her sleep. The pain is improved by activity but not relieved by resting. She finds it difficult to bend down during the day to pick things up from the floor.
Which clinical feature is most specific for inflammatory back pain?
A. Improvement with activity
B. Nocturnal pain
C. Radiation to leg
D. Stiffness during the day
E. Young age

A

A. Improvement with activity

285
Q

A 68 year old man has a swollen tender knee for 3 days and cannot weight bear. He has had previous episodes of big toe swelling. He has a history of chronic kidney disease stage 4.
Investigations:
Fluid analysis of knee aspirate:
White cell count 55 000/mL, 95% neutrophils Gram stain negative
Copious 10 μm intracellular needle shaped crystals
Which is the best initial treatment for his acute knee pain and swelling?
A. Arthroscopic joint washout
B. Intravenous flucloxacillin
C. Oral allopurinol
D. Oral naproxen
E. Oral prednisolone

A

E. Oral prednisolone

286
Q

An 18 year old man is in the intensive care unit with septic shock as a result of a compound fracture of his leg. His urine output has been <30 mL for the past 2 hours.
His pulse rate is 125 bpm, BP 85/40 mmHg and JVP 4 cm above the sternal angle. There are basal crackles on auscultation of the chest.
Investigations:
ECG shows sinus rhythm, 125 bpm.
He is being treated with intravenous 0.9% sodium chloride at 125 mL/h and antibiotics.
Which is the most appropriate additional intravenous treatment?
A. 500 mL 0.9% sodium chloride over 15 min
B. 500 mL Hartmann’s solution over 15 min
C. Dopamine hydrochloride
D. Furosemide
E. Noradrenaline/norepinephrine

A

E. Noradrenaline/norepinephrine

287
Q

A 34 year old man has pain in his right shoulder and upper arm for 6 weeks that worsens when elevating his arm above his head. He does not recall any injury.
There is no deformity, tenderness or reduced range of movement. There is pain on abduction of the right shoulder that is worse with the arm in internal rotation and when abduction is resisted. He is treated with ibuprofen.
Which is the most appropriate next step in management?
A. MR scan of shoulder
B. Refer for orthopaedic opinion
C. Refer for physiotherapy
D. Ultrasound scan of shoulder
E. X-ray of shoulder

A

C. Refer for physiotherapy

288
Q

A 65 year old woman with advanced carcinoma of the breast with cerebral and liver metastases is rapidly deteriorating and is recognised to be dying. She becomes confused and agitated.
Which is the most suitable initial treatment?
A. Diamorphine hydrochloride
B. Glycopyrronium
C. Hyoscine hydrobromide
D. Midazolam
E. Mirtazapine

A

D. Midazolam

289
Q

A 80 year old man has progressive memory loss over 2 years.
He scores 20/30 on the Montreal Cognitive Assessment, and he requires an MR
scan of brain.
In which part of the brain are changes most likely to be found in early Alzheimer’s disease?
A. Basal ganglia
B. Frontal lobe
C. Medulla oblongata D. Parietal lobe
E. Temporal lobe

A

E. Temporal lobe

290
Q

A 43 year old woman is admitted to hospital with an overdose of amitriptyline hydrochloride taken 4 hours previously. She has a history of chronic headache.
Her pulse rate is 105 bpm and BP 95/40 mmHg. She is drowsy and her pupils are dilated. ECG shows significant QRS prolongation.
Which is the most appropriate immediate management option?
A. Activated charcoal
B. Intravenous amiodarone hydrochloride
C. Intravenous atropine sulfate
D. Intravenous magnesium
E. Intravenous sodium bicarbonate

A

E. Intravenous sodium bicarbonate

291
Q

A 76 year old man undergoes an abdominoperineal resection for a low rectal carcinoma. He has a history of severe COPD and hypertension.
Which is the most appropriate method for providing analgesia during the early postoperative period?
A. Epidural anaesthesia
B. Intramuscular opioid
C. Oral non-steroidal analgesia
D. Patient controlled intravenous analgesia E. Spinal anaesthesia

A

A. Epidural anaesthesia

292
Q

A 75 year old woman becomes breathless while receiving the third unit of a blood transfusion. She was admitted with melaena and a haemoglobin of 65 g/L (115– 150). She has a history of ischaemic heart disease and takes aspirin and lisinopril.
She now has bibasal inspiratory crackles and occasional wheeze. Her temperature is 37.3°C, pulse rate 96 bpm, BP 120/80 mmHg, respiratory rate 30 breaths per minute and oxygen saturation 90% breathing air.
She is treated with high flow oxygen. The nursing staff have stopped the blood transfusion.
Which is the most appropriate additional treatment?
A. Intramuscular adrenaline/epinephrine
B. Intravenous chlorphenamine maleate
C. Intravenous furosemide
D. Intravenous hydrocortisone
E. Nebulised salbutamol

A

C. Intravenous furosemide

293
Q

A researcher is seeking to examine whether long-term mobile phone use is linked to acoustic neuroma risk. The information on mobile phone usage is collected from participants with acoustic neuroma and a comparable group of participants without acoustic neuroma, selected from the general practice register.
Which type of study design is being used?
A. Case–control study
B. Case series
C. Cohort study
D. Cross-sectional study
E. Ecological study

A

A. Case–control study

294
Q

A 56 year old woman has home blood pressure readings averaging 160/90 mmHg.
Hypertension is confirmed on 24 hour ambulatory monitoring. She has type 1 diabetes mellitus.
Investigations:
Urinary albumin: creatinine ratio 42 mg/mmol eGFR 43 mL/min/1.73 m2
Which type of antihypertensive is most appropriate?
A. ACE inhibitor
B. Alpha blocker
C. Beta blocker
D. Calcium channel blocker
E. Thiazide-like diuretic

A

A. ACE inhibitor

295
Q

A 70 year old man has dry cough and breathlessness on exertion for the past 3 months. He has lost 4 kg in weight. He has a history of ischaemic heart disease and atrial fibrillation. He takes warfarin sodium, ramipril and amiodarone hydrochloride. He is a never smoker.
His temperature is 37.5°C, pulse rate 70 bpm, respiratory rate 18 breaths per minute and oxygen saturation 91% breathing air. He has fine bibasal inspiratory crackles. There is no finger clubbing.
Investigations:
Haemoglobin
White cell count
Erythrocyte sedimentation rate
141 g/L (130–175) 14.0 × 109/L (3.0–10.0)
65 mm/hr (<20)
Chest X-ray shows bilateral reticular opacities in both bases.
Which investigation is most likely to confirm the diagnosis?
A. Blood cultures
B. Bronchoscopy
C. Echocardiography
D. High resolution CT scan of chest
E. Induced sputum for microscopy and culture

A

D. High resolution CT scan of chest

296
Q

A 31 year old woman is admitted with 24 hours of confusion. She is having visual hallucinations of snakes and mice on the floor. Her partner says that she often drinks 80 units of alcohol per week. He has not seen her for the past week. She has a history of depression and takes fluoxetine. Her temperature is 37.6°C, pulse rate 100 bpm and BP 162/98 mmHg. She is disorientated in time and place. She has no focal neurological deficit.
Which is the most likely diagnosis?
A. Delirium tremens
B. Fluoxetine overdose
C. Hepatic encephalopathy
D. Korsakoff’s psychosis
E. Wernicke’s encephalopathy

A

A. Delirium tremens

297
Q

A 61 year old woman is admitted with 2 days of confusion. She has a history of hypertension and takes nifedipine. She smokes 20 cigarettes per day. She is confused but has no focal neurological deficit. Her pulse rate is 75 bpm, BP 139/87 mmHg and JVP 2 cm above the sternal angle.
Investigations: Sodium Potassium Urea Creatinine
117 mmol/L 4.2 mmol/L 1.9 mmol/L
57 μmol/L
(135–146) (3.5–5.3) (2.5–7.8) (60–120)
(285–295) (100–1000)
Serum osmolality 252 mOsmol/kg Urine osmolality 585 mOsmol/kg
Which mechanism best explains the development of hyponatraemia?
A. Increased sodium secretion in the distal tubule
B. Increased water absorption in the collecting duct
C. Increased water ingestion
D. Reduced cortisol secretion
E. Reduced sodium reabsorption in the proximal tubule

A

B. Increased water absorption in the collecting duct

298
Q

A 43 year old woman has a sudden severe headache that started 24 hours ago. She has a history of autosomal dominant polycystic kidney disease.
Her temperature is 36.8°C, pulse rate 92 bpm and BP 140/100 mmHg. Neurological examination is normal. CT scan of head is normal.
Which is the most appropriate next step in management?
A. Erythrocyte sedimentation rate
B. Lumbar puncture
C. MR scan of brain
D. Refer to outpatient headache clinic
E. Start amlodipine

A

B. Lumbar puncture

299
Q

A 74 year old man has progressively worsening muscle aches that are now causing him to struggle to get up from a chair or raise his arms above his head. He has a history of oesophageal cancer treated surgically and ischaemic heart disease. He is taking bisoprolol, clopidogrel, ramipril and simvastatin.
Investigations:
Haemoglobin
White cell count
Platelets
Erythrocyte sedimentation rate 105 mm/hr
(130–175) (3.0–10.0) (150–400)
(<20)
391 × 109/L Which is the most appropriate therapeutic change?
125 g/L 7.8 × 109/L
A. Start co-codamol
B. Start ibuprofen
C. Start prednisolone
D. Stop bisoprolol
E. Stop simvastatin

A

C. Start prednisolone

300
Q

A 59 year old woman has 6 months of pain affecting her hips and lower back. She is Libyan and has lived in the UK for 10 years. She has chronic kidney disease stage 3 and hypertension. She is taking lisinopril and simvastatin. She has weakness of hip flexion bilaterally. There is no muscle or bony tenderness.
Investigations:
Urea 7.8 mmol/L Creatinine 122 μmol/L Calcium 2.1 mmol/L
eGFR 41 mL/min/1.73 m2
Alkaline phosphatase 230 IU/L
Parathyroid hormone 14.5 pmol/L
Which additional investigation is most likely to confirm the diagnosis?
A. Creatine kinase
B. Erythrocyte sedimentation rate
C. Serum 25-OH cholecalciferol
D. Ultrasound scan of neck
E. X-ray of thoracic and lumbar spine

A

C. Serum 25-OH cholecalciferol